PACKRAT GI

Réussis tes devoirs et examens dès maintenant avec Quizwiz!

For patients undergoing highly emetogenic chemotherapy regimens, which of the following would be the first-line antiemetic medication? A. Ondansetron (Zofran) B. Lorazepam (Ativan) C. Dronabinol (Marinol) D. Prochlorperazine (Compazine)

(c) A. A 5HT3 receptor blocker such as ondansetron is the agent of choice, given its high efficacy and low risk of side effects (headache being the major, and usually easily manageable, one). (u) B. Benzodiazepines (lorazepam) are associated with significant psychologic side effects including sedation and amnesia, although its anxiolytic properties make it a good choice for patients suffering from anticipatory nausea. (u) C. Cannabinoids (dronabinol) likewise have significant psychogenic side effects. (u) D. The side effect profile of phenothiazides (prochlorperazine) includes hypotension, restlessness, sedation, and extrapyramidal symptoms.

A 68 year-old male presents with jaundice, weight loss, and boring abdominal pain which radiates to the back. The gallbladder is palpable on physical examination. This finding is most consistent with which of the following? A. Pancreatic tumor B. Hemorrhagic pancreatitis C. Cholecystitis D. Cholelithiasis

(c) A. A large palpable gallbladder resulting from pressure from a tumor in the pancreatic head is known as Courvoisier's sign. (u) B. Ecchymosis of the flank associated with hemorrhagic pancreatitis is known as Grey Turner's sign (u) C. A sharp increase in tenderness with a sudden stop in inspiratory effort indicative of acute cholecystitis is known as Murphy's sign. (u) D. Pain in the right lower quadrant during left-sided pressure on the abdomen suggesting acute appendicitis is known as Rovsing's sign.

Which of the following is the most appropriate study for diagnosing Hirschsprung disease? A. Rectal biopsy B. Stool leukocyte test C. CT of the abdomen and pelvis D. Fecal occult blood test

(c) A. A rectal biopsy showing the absence of ganglion cells in both the submucosal and muscular layers of the involved bowel is the most appropriate diagnostic study for Hirschsprung disease. (u) B. Stool leukocyte testing can indicate an infectious etiology of diarrhea and is not indicated in the diagnosis of Hirschsprung disease. (u) C. Radiographic examination may show dilated proximal colon and absence of gas in the pelvic colon, but is not diagnostic for Hirschsprung disease. (u) D. Fecal occult blood testing is not indicated in the diagnosis of Hirschsprung disease.

What is considered the most common physical examination finding for intestinal obstruction? A. Distention B. Fluid wave C. Rigidity D. Tenderness

(c) A. Abdominal distention is the most common hallmark of all kinds of intestinal obstructions though its presence can be variable depending on the duration and exact location of the obstruction. (u) B. Fluid wave is noted with ascites, not intestinal obstruction. (u) C. Rigidity is most often minimal to absent except in the most extreme of late presenting strangulated obstructions and is more commonly found early in the course of peritonitis and related phenomenon. (u) D. Tenderness, like rigidity, is not a predominant finding in obstructed bowel until very late in the course of advancing, untreated cases. At this time distention and symptoms of shock will also be present and command the clinical picture.

A 45 year-old male nonsmoker presents with a 3-month history of difficulty swallowing. He describes the gradual onset of dysphagia for solids and liquids; he now complains of substantial discomfort after eating. What is his most likely diagnosis? A. Achalasia B. Schatzki ring C. Esophageal carcinoma D. Zenkers diverticulum

(c) A. Achalasia is by far the most common cause of dysmotility. This is particularly true in a non-smoker (less likely to be esophageal carcinoma or a lung cancer causing a paraneoplastic syndrome) and someone who has not been to an area endemic for Chagas disease. (u) B. See A for explanation. (u) C. See A for explanation. (u) D. See A for explanation.

A 20 year-old male presents with a mass in the groin. On examination with the patient standing, a mass is noted that extends into the scrotum. The patient denies any trauma. The most likely diagnosis is A. an indirect inguinal hernia. B. a direct inguinal hernia. C. an obturator hernia. D. a femoral hernia.

(c) A. An indirect inguinal hernia is caused by a patent processus vaginalis and the hernial contents may be felt in the ipsilateral scrotum. (u) B. A direct inguinal hernia is symmetrical, round and disappears easily with the patient lying down. It is the result of a weakness in the inguinal 3external ring. Hernial contents may radiate anteriorly rather than into the scrotum. (u) C. Obturator hernia's are more commonly seen in elderly women and are rarely palpable in the groin. (u) D. Femoral hernias are rare in males and do not typically reduce with lying down.

A patient presents uncertain as to whether he has completed the Hepatitis B vaccination series. What laboratory test would be most helpful in determining his immunization status? A. Anti-HBs B. HBeAg C. HBsAg D. Anti-HBc

(c) A. Anti-HBs is the circulating antibody that develops to surface antigen in response to either past Hepatitis B infection or to Hepatitis B immunization. It represents immunity to the Hepatitis B virus in otherwise immunocompetent patients. (u) B. HBeAg is seen with active disease and not from immunization. (u) C. The surface antigen to Hepatitis B (HBsAg) is most often used to show acute infection and not immunity. (u) D. Anti-HBc is seen with active disease and not from immunization.

Which of the following symptoms most commonly indicates diarrhea of an inflammatory etiology? A. Bloody stools B. Large volume stools C. Fatty stools D. Watery stools

(c) A. Bloody diarrhea occurs due to mucosal inflammation and/or erosions/ulcerations and may be caused by infection, inflammation, or ischemic enterocolitis. (u) B. Large volume indicates anatomical orgin (small intestine) but not etiology. (u) C. Fatty stools usually seen in malabsorptive conditions of the small intestine and do not necessarily involve inflammation but the increase in fat remaining in the lumen. (u) D. Watery stools are just an indication of increase amount of water, either osmotic or secretory in nature.

A 24 year-old female presents with complaints of abdominal pain, bloating, and chronic diarrhea. She says she felt worse last month while on vacation in Italy. Despite eating well she lost weight. What would be the most appropriate treatment for this patient? A. Gluten-free diet B. Small bowel resection C. Ciprofloxacin (Cipro) therapy D. Pancreatic enzymes

(c) A. Celiac disease is a diffuse disease of the small bowel that is caused by immunologic malfunction. This disease is active only in the presence of gluten, a constituent of wheat. Avoidance of gluten-containing foods is the treatment of choice. (u) B. Although celiac disease affects the small bowel (particularly the jejunum), resection is too aggressive when dietary manipulation is very effective in the management of this disorder. (u) C. Celiac disease is immune-mediated and is not due to infection so antibiotics are not needed. (u) D. Celiac disease affects absorption in the small bowel and does not involve a dysfunction of pancreatic enzymes.

A 30 year-old patient presents with weight loss, diarrhea, and steatorrhea. Labs reveal that the antiendomysial antibody (AEA) is positive. What is the most likely diagnosis? A. Celiac sprue B. Ulcerative colitis C. Whipple's disease D. Zollinger-Ellison syndrome

(c) A. Celiac sprue is not only characterized by these classic symptoms. The antiendomysial antibody has a 90-95% sensitivity and 90-95% specificity for celiac sprue. (u) B. Ulcerative colitis could be responsible for the symptoms mentioned however antiendomysial antibody (AEA) would not be positive. (u) C. Whipple's disease is an infectious disorder known to cause diarrhea and weight loss. It also results in arthralgia and symptoms involving both the central nervous and cardiac systems that are not easily missed due to their severity. Diagnosis involves biopsies of the involved tissues looking for PAS-positive macrophages. (u) D. Zollinger-Ellison syndrome (ZE) is a hypersecretory disorder. Diarrhea can occur but will not generally be the predominant symptom.

What is the term for blue discoloration about the umbilicus? A. Cullen's sign B. Murphy's sign C. Rovsing's sign D. Turner sign

(c) A. Cullen's sign is a blue discoloration about the umbilicus and can occur in hemorrhagic pancreatitis and results from hemoperitoneum. (u) B. Murphy's sign is seen in liver and gallbladder disease by which the patient abruptly halts deep inspiration due to pain in the right upper quadrant while the examiner's hand is keeping stable pressure to the area. (u) C. Rovsing's sign is positive when the patient experiences right sided abdominal pain with palpation to the left side. This is generally seen in those with appendicitis. (u) D. Turner sign is also seen in severe, acute pancreatitis but is represented by a green-brown discoloration of bilateral flanks.

A 68 year-old male with a history of alcohol abuse presents with coffee-ground emesis. He denies vomiting prior to this episode. What is the most likely cause of his bleeding? A. Erosive gastritis B. Gastric neoplasm C. Mallory-Weiss tear D. Zenker diverticulum

(c) A. Drugs and alcohol are the most common causes of upper gastrointestinal bleeding. (u) B. Gastric neoplasm is also uncommon (less than 1% of upper GI bleeding) but should be ruled out). (u) C. Mallory-Weiss tears are lacerations of the gastroesophageal junction causing 5-10% of cases of upper GI bleeding. Many patients report a history of heavy alcohol use or retching. (u) D. Peptic ulcer disease is a common cause of upper GI bleeding but is becoming less common with the increasing irradication of H. pylori infections.

A 65 year-old patient with known history of alcohol and tobacco abuse presents with solid food dysphagia. The patient also has a 24 lb weight loss over the past 6 months. Which of the following is the most appropriate intervention? A. endoscopy with biopsy B. chest x-ray C. barium esophagogram D. CT scan of the thorax

(c) A. Endoscopy with biopsy establishes the diagnosis of esophageal cancer with a high degree of reliability. (u) B. Chest x-ray may show adenopathy, a pulmonary or bony metastases or sign of tracheoesophageal fistula. Barium esophagogram is obtained as the first study to evaluate the dysphagia. CT scan should be obtained once the diagnosis of carcinoma has been made to evaluate for pulmonary or hepatic metastases, lymphadenopathy, and local tumor extension. None of these tests will reveal the diagnosis of carcinoma. (u) C. See B for explanation. (u) D. See B for explanation.

A 62 year-old male is brought to the emergency department with acute hematemesis. The patient denies a previous history of vomiting. His wife states he has chronic liver disease. Physical examination reveals a distended abdomen without rebound, guarding or organomegaly. There is a fluid wave. Which of the following is the most likely diagnosis? A. Esophageal varices B. Mallory-Weiss tear C. Arteriovenous malformation D. Perforated duodenal ulcer

(c) A. Esophageal varices are dilated submucosal veins that develop in a patient with underlying portal hypertension. The most common cause of portal hypertension is cirrhosis. (u) B. A patient with a Mallory-Weiss tear would have a history of retching but would not have a distended abdomen. (u) C. Most arteriovenous malformations are asymptomatic. If symptomatic they would have symptoms of a slow bleed. (u) D. A patient with perforated duodenal ulcer would have rebound and guarding on examination.

A 35 year-old female 2 months status-post gastric bypass surgery develops abdominal cramps, diarrhea, palpitations, sweating and nausea shortly after eating. Which of the following is the most likely diagnosis? A. Dumping syndrome B. Inflammatory bowel disease C. Surgical adhesions D. Zollinger-Ellison syndrome

(c) A. Gastric bypass puts this patient at risk for dumping syndrome. Her symptoms represent both cardiovascular and gastrointestinal reactions which occur after eating. The underlying effect is due to the stomach's inability to regulate its rate of emptying. (u) B. IBD does not present in acute attacks after meals and usually does not present with autonomic symptoms. (u) C. Small bowel obstruction due to peritoneal adhesions is a possibility due to her history of abdominal surgery. However, obstruction presents with abdominal pain, nausea and vomiting which may progress and are not associated with meals. (u) D. A gastrinoma leads to PUD refractory to large amounts of antacids and standard doses of H2 blockers

A 31 year-old female complains of diarrhea. She admits to associated weight loss and steatorrhea, but denies melena or hematochezia. Which of the following is the most likely diagnosis? A. Celiac disease B. Lactose intolerance C. Short bowel syndrome D. Irritable bowel syndrome

(c) A. Greater than 10 grams of fecal fat in 24 hours is most consistent with a malabsorption syndrome such as celiac sprue. (u) B. See A for explanation. (u) C. See A for explanation. (u) D. See A for explanation.

Which of the following is an indication for vaccination against hepatitis A? A. Illicit drug users B. Health care workers C. Renal dialysis patients D. Routine vaccination starting at birth

(c) A. Hepatitis A vaccine is recommended for illicit drug users, anyone living or traveling to endemic areas, sewage workers, food handlers, homosexual and bisexual men, animal handlers, patients with a history of chronic liver disease or a clotting factor disease as well as children and workers in day care settings and institutions. (u) B. Health care workers, renal dialysis patients and routine vaccination starting at birth are some of the recommendations for vaccination against hepatitis B, not hepatitis A. (u) C. See B for explanation. (u) D. See B for explanation.

Congenital absence of ganglionic nerve cells innervating the bowel wall is seen in which of the following conditions? A. Hirschsprung's disease B. Meckel's diverticulum C. Chagas disease D. Hashimoto's hypothyroidism

(c) A. Hirschsprung disease, also termed congenital aganglionic megacolon, results from a lack of ganglion cells in the bowel wall. (u) B. Meckel's diverticulum results from the failure of the vitelline duct to separate from the intestine during early gestation leading to an outpouching in the intestinal tract. (u) C. Chagas disease results from infection with the Trypanosoma parasite. Chronic Chagas disease may lead to the loss of dorsal motor nuclear cells of the vagus nerve, resulting in megaesophagus and megacolon. (u) D. Ineffective peristalsis of the colon from hypothyroidism may result in constipation but not Hirschsprung disease

A 56 year-old male with a history of hypertension and alcohol use presents with mid-epigastric pain and coffee-ground emesis since early this morning. The patient denies diarrhea, constipation or blood in his stools. Endoscopic evaluation done after admission revealed peptic ulcer disease with active bleeding. Which of the following is the most appropriate therapy to reduce the risk of rebleeding in this patient? A. IV pantoprazole (Protonix) B. Oral hyoscyamine sulfate (Levsin) C. IV famotidine (Pepcid) D. Oral metoclopramide (Reglan)

(c) A. IV proton pump inhibitors reduce the risk of rebleeding after endoscopic treatment in patients with peptic ulcers that have high risk features such as active bleeding. (u) B. Hyoscyamine is an anticholinergic used as an adjunct in the treatment of peptic ulcer disease and to treat irritable bowel syndrome. It does not have a role in reducing the risk of rebleeding in these patients. (u) C. H2 antagonists do not stop active bleeding and do not reduce the incidence of rebleeding. (u) D. Metoclopramide is used in the management of GERD and nausea and is not indicated in the reduction of rebleeding secondary to peptic ulcer disease.

To further assess ascites in a patient, the physician assistant instructs the patient to turn onto one side while performing percussion. Which of the following is the reason for this maneuver? A. Testing for shifting of dullness on percussion B. Shifting of internal organs making percussion easier C. Trying to elicit any pain while moving D. Trying to produce a caput medusa

(c) A. In ascites, dullness shifts to the more dependent side as the fluid relocates into dependent space, while tympany shifts to the top as the gas-filled organs float to the top of the ascitic fluid. (u) B. See A for explanation. (u) C. Pain with movement is associated with peritonitis and not ascites. (u) D. Caput medusa is the dilation of the superficial abdominal veins due to increased intraabdominal fluid accumulation. It is visible with the patient standing and does not need a special maneuver for identification.

Which of the following is consistent with acute cholangitis? A. Jaundice B. Caput medusa C. Bilateral flank bruising D. An enlarged, palpable nontender gallbladder

(c) A. Jaundice is part of Charcot's triad associated with cholangitis along with fever and biliary colic. (u) B. Caput medusa is associated with ascites and cirrhosis. (u) C. Bilateral flank bruising is associated with hemorrhagic pancreatitis. (u) D. An enlarged, nontender, palpable gallbladder is associated with cancer of the head of the pancreas.

Initial pharmacologic treatment of acute hepatic encephalopathy consists of A. lactulose. B. omega-3-fatty acids. C. neomycin. D. mannitol.

(c) A. Lactulose acts as an osmotic laxative decreasing ammonia absorption and decreases ammonia production by directly affecting bacterial metabolism. (u) B. Omega-3-fatty acids have no role in the treatment of hepatic encephalopathy. (u) C. Neomycin, like lactulose, will decrease ammonia production by intestinal bacteria and may have a role in chronic but not acute management of hepatic encephalopathy. (u) D. Mannitol is used as an osmotic diuretic

An otherwise healthy 23 year-old female presents to the student health office complaining of 3 days of frequent, watery, non-bloody stools. She denies significant abdominal pain, vomiting, fever or dark urine. Others on campus have been seen with the same presentation this week. Initial choice of treatment includes which of the following? A. Fluid intake and bowel rest B. Ciprofloxacin C. Hospital admission with IV fluids D. Atropine

(c) A. Most mild diarrhea will not lead to dehydration with adequate fluids and comfort with rest to the bowel. (u) B. Empiric treatment with antibiotics is not indicated this early. (u) C. See A for explanation. (h) D. Atropine is contraindicated as an antidiarrheal due to the possibility of toxic megacolon.

The first step in the treatment of a patient with an intestinal obstruction and no comorbid diseases is A. nasogastric decompression. B. invasive hemodynamic monitoring. C. abdominal exploration. D. administration of antibiotics.

(c) A. Nasogastric decompression is indicated in all but mild cases of obstruction to prevent distal passage of swallowed air and minimize distension. (u) B. Invasive hemodynamic monitoring is needed only if the patient has underlying cardiac, pulmonary, or renal disease. (u) C. Abdominal exploration is considered only if the obstruction does not resolve in 24 to 48 hours or if peritoneal findings, fever, or rapidly progressing abdominal pain occur. (u) D. Antibiotics are given only if surgery is to be done.

A 30 year-old female presents with a one to two year history of daily bouts of mid-abdominal crampy pain and bloating that are briefly relieved with defecation. Loose to watery bowel movements occur four to five times daily. She denies any nocturnal symptoms. Previous laboratory studies, abdominal CT scans and a colonoscopy have all been normal. Attempted dietary changes, exercise and probiotics have provided no relief. What is the next step in treatment for this patient? A. Nortriptyline (Pamelor) B. Omeprazole (Prilosec) C. Polyethylene glycol (Miralax) D. Promethazine (Phenergan)

(c) A. Nortriptyline, and other tricyclic antidepressants (TCAs), is an appropriate choice for someone with diarrhea predominant irritable bowel syndrome due to their numerous mechanisms of action. (u) B. Omeprazole, a proton pump inhibitor, works on parietal cells in the stomach lining to reduce production of hydrochloric acid. Though beneficial for peptic ulcer and reflux disease it would provide no relief of symptoms in a patient with IBS. (u) C. Polyethylene glycol is commonly used in constipation predominant IBS but would probably worsen this patient's symptoms. (u) D. The anticholinergic properties of the promethazine might be helpful even though its somnolent effects would be significant and would not be superior to nortriptyline. Promethazine's strength is in nausea control and less often its antihistamine affects.

Which of the following pathophysiological processes is believed to initiate acute appendicitis? A. Obstruction B. Perforation C. Hemorrhage D. Vascular compromise

(c) A. Obstruction of the appendiceal lumen by lymphoid hyperplasia, a fecalith or foreign body initiates most cases of appendicitis. (u) B. See A for explanation. (u) C. See A for explanation. (u) D. See A for explanation.

Gallstones usually result in biliary symptoms by causing inflammation or obstruction following migration into the common bile duct or A. cystic duct. B. pancreatic duct. C. duodenal ampulla. D. common hepatic duct.

(c) A. Obstruction of the cystic duct by gallstones causes the typical symptom of biliary colic. Once obstructed the gallbladder distends and becomes edematous and inflamed. Gallstones can also migrate into the common bile duct through the cystic duct leading to a condition known as choledocholithiasis. (u) B. Obstruction of the pancreatic duct leads to development of acute pancreatitis. (u) C. The duodenal ampulla is the area where the pancreatic duct and the common bile duct empty into the duodenum. Gallstones do not cause obstruction at this distal site. (u) D. The common hepatic duct from the liver joins the cystic duct from the gallbladder to form the common bile duct.Stone migration occurs along the pathway of the cystic duct to the common bile duct, not along the common hepatic duct.

A 52 year-old female comes to the office because of black stools for the past 3 days. She is afebrile and she has no pertinent physical examination abnormalities. Which of the following is the most appropriate initial diagnostic study? A. Stool for occult blood B. Stool cultures C. Sigmoidoscopy D. Abdominal CT scan

(c) A. Occult bleeding, as evidenced by the patient's history of black stools, is initially verified by a positive fecal occult blood test. (u) B. Stool cultures are indicated in the evaluation of acute diarrhea and not for the evaluation of acute GI bleeding. (u) C. Melena suggests a source of bleeding that is proximal to the ligament of Treitz, not a lower GI bleed. Sigmoidoscopy is used to evaluate only lower GI bleeding sources. (u) D. Abdominal CT scan is indicated for evaluation of obscure bleeding in order to exclude a pancreatic or hepatic source of bleeding if endoscopy fails to identify the source.

A 45 year-old female presents with bowel movements associated with bright red blood after being diagnosed with a stage II hemorrhoid 1 month ago. She has used increased fiber and fluid intake as her primary treatment up to this point. Which of the following is the most appropriate next step in managing this patient's condition? A. Rubber band ligation B. Hemorrhoidectomy C. Diltiazem (Cardizem) therapy D. Iron supplementation

(c) A. Patients who have a stage I, II or III hemorrhoid with recurrent bleeding not responsive to conservative therapy should have injection sclerotherapy, rubber band ligation or application of electrocoagulation. Rubber band ligation is easy to use and has a high efficacy rate. (u) B. Hemorrhoidectomy is indicated in patients with a stage III or IV hemorrhoid with chronic bleeding or a stage II hemorrhoid that is acutely thrombosed. (u) C. Diltiazem is not indicated in the management of hemorrhoids and may exacerbate the patient's symptoms by causing constipation. (u) D. There is rarely significant bleeding secondary to hemorrhoid disease that necessitates iron supplementation. Additionally, this will not treat the underlying disorder.

A patient was treated for community acquired pneumonia with amoxicillin-clavulanate (Augmentin). On day 7 of therapy he develops fulminate diarrhea. The diarrhea is described as greenish and foul-smelling. He admits to associated abdominal cramps. Which of the following is the treatment of choice for this patient? A. Metronidazole (Flagyl) B. Diphenoxylate/atropine (Lomotil) C. Clindamycin (Cleocin) D. Ciprofloxacin (Cipro)

(c) A. Patients with C. difficile colitis should be treated with Flagyl for 10-14 days following cessation of the diarrhea-inducing antibiotics. (u) B. See A for explanation. (u) C. See A for explanation. (u) D. See A for explanation.

A 45 year-old female presents with a ten pound weight loss and recurrent greasy stools mixed with diarrhea. The patient notes that these symptoms are worse with certain foods. Which of the following laboratory tests should initially be ordered? A. anti-endomysial antibodies B. anti-mitochondrial antibodies C. anti-glomerular basement membrane antibodies D. anti-phospholipid antibodies

(c) A. Patients with celiac sprue disease are likely to have anti-endomysial antibody formation. (u) B. Anti-mitochondrial antibodies are present with hypothyroid disease and other autoimmune disease processes. (u) C. Anti-glomerular basement disease occurs in the kidney and lungs and these antibodies are positive in patients with Goodpasture's disease. (u) D. Anti-phospholipid antibodies are seen in patients who have systemic lupus erythematosus.

A 3 year-old presents with a 24-hour history of diarrhea. The patient is afebrile and the stool is noted to be loose and watery. No blood is noted in the stool. Fecal WBC is negative. Which of the following is the most likely diagnosis? A. viral gastroenteritis B. toxic megacolon C. ulcerative colitis D. intussusception

(c) A. Patients with viral gastroenteritis are afebrile and noted to have loose, watery, non-bloody diarrhea. Fecal WBC is negative. (u) B. Toxic megacolon is a complication of ulcerative colitis and presents with diarrhea, fever, tachycardia, and leukocytosis. (u) C. Ulcerative colitis presents with bloody diarrhea, abdominal pain, and fever. (u) D. Intussusception is more common in children age 1-2 years and presents with crampy abdominal pain, vomiting, and currant jelly stool.

A 48 year-old male presents with complaints of heartburn that occurs approximately 45 minutes after eating about three times a week that is relieved by antacids. He claims to have followed advice about elevating the head of the bed, avoiding spicy foods, and losing weight, but continues to have heartburn. Which of the following is the most appropriate next step? A. Ranitidine (Zantac) B. Sucralfate (Carafate) C. Metoclopramide (Reglan) D. Misoprostol (Cytotec)

(c) A. Ranitidine, an H2 receptor blocker, is indicated for the treatment of mild, intermittent symptoms of gastroesophageal reflux disease. (u) B. Sucralfate is used in the treatment of duodenal ulcers. (u) C. Metoclopramide is indicated for the treatment of gastroparesis as a first-line agent and as a second-line agent in the treatment of refractory gastroesophageal reflux. (u) D. Misoprostol is indicated for the prevention of NSAID-induced gastritis.

A 53 year-old man who had previously been in good health presents to the emergency department with a history of passing approximately 300 cc of bright red blood from the rectum 45 minutes ago. From which of the following anatomical sites is the bleeding most likely originating? A. rectosigmoid B. duodenum C. stomach D. esophagus

(c) A. Rectosigmoid is the most common site leading to hematochezia. (u) B. See A for explanation. (u) C. See A for explanation. (u) D. See A for explanation.

Which of the following is the treatment of choice for patients with celiac disease? A. Gluten-free diet B. Small bowel resection C. Clindamycin D. Whipple procedure

(c) A. Removal of all gluten from the diet is essential to therapy in celiac disease. (u) B. Celiac disease is a dietary disorder that resolves with gluten free diet. Small bowel resection is not indicated. (u) C. Antibiotics are not therapeutic for celiac disease. (u) D. Whipple procedure is used in the treatment of pancreatic carcinoma and is not indicated in the management of celiac disease.

Which of the following signs is positive in the presence of ascites? A. shifting dullness B. CVA tenderness C. ballottment sign D. cutaneous hypersensitivity

(c) A. Shifting dullness or the presence of a fluid wave is noted in patients with ascites. (u) B. CVA tenderness is noted in patients with pyelonephritis. (u) C. Ballottment test is used to assess a floating mass in the abdomen. (u) D. Cutaneous hypersensitivity is used to identify a specific zone of peritoneal irritation.

Which of the following is the laboratory test that marks recovery from Hepatitis B infection and non-infectivity? A. Hepatitis B surface antibody(anti-HBs) B. Hepatitis B surface antigen (HBsAg) C. Hepatitis B core antigen (HBcAg) D. Hepatitis A antibody (anti-HAV)

(c) A. Specific antibody to HBsAg appears in most individuals after clearance of HBsAg which indicates recovery from hepatitis B infection, non-infectivity, and immunity. (u) B. This test establishes infection with HBV and implies infectivity. (u) C. Presence of the hepatitis B core antigen reflects active infection. (u) D. Patients who have immunity to hepatitis A do not have immunity to hepatitis B.

Which of the following is the most important intervention in acute pancreatitis? A. IV fluid administration B. Antibiotic administration C. Calcium replacement D. Antiemetics

(c) A. The mainstay of management in acute pancreatitis is fluid resuscitation. Isotonic solutions are best to maintain renal perfusion and urine output > 100 ml/hour. (u) B. 90% of patients will recover with supportive measures only. Parenternal antibiotics are not indicated in acute pancreatitis. (u) C. Hypocalcemia is a marker of prognosis, but correction of hypocalcemia does not supercede the importance of fluid resuscitation from the overwhelming inflammatory response. (u) D. Other supportive measures in acute pancreatitis include pain management and antiemetics, but the mainstay is fluid resuscitation.

A 68 year-old woman presents with intermittent crampy abdominal pain and vomiting of 6 hours' duration. She feels bloated but denies rectal bleeding. Examination of the abdomen reveals no palpable mass and you note an upper midline scar from previous abdominal surgery. An abdominal radiograph demonstrates dilated loops of small bowel with air-fluid levels. Which of the following is the most likely cause of this patient's symptoms? A. Adhesions B. Sigmoid volvulus C. Incisional hernia D. Intussusception

(c) A. The patient's surgical history, presenting symptoms and abdominal imaging are consistent with small bowel obstruction. Peritoneal adhesions are the most common cause of small bowel obstruction. (u) B. Sigmoid volvulus or twisting of the sigmoid colon on its mesentery leads to colonic obstruction. This patient does not complain of constipation or report laxative use. Abdominal radiographs would show a dilated colon lacking haustral folds. (u) C. Hernias may occur after abdominal surgery but are usually more chronic in nature and a mass would be palpable especially with maneuvers that increase intra-abdominal pressure. (u) D. Intussusception, although rare in adults, may occur due to small bowel neoplasia but this patient does not complain of bleeding and no mass was palpable on exam. Also a CT scan would usually show a target lesion representing the layers of intussuscepted segment.

A 40 year-old female complains of acute right upper quadrant pain radiating to the back and low grade fever. Laboratory evaluation indicates the presence of urinary bilirubin and an elevation of serum alkaline phosphatase. Which of the following is the most likely diagnosis? A. cholecystitis B. viral hepatitis C. Gilbert's syndrome D. Dubin-Johnson syndrome

(c) A. The presence of urinary bilirubin indicating conjugated hyperbilirubinemia coupled with the elevation of serum alkaline phosphatase suggests biliary obstruction that may lead to cholecystitis. (u) B. Although viral hepatitis presents with conjugated hyperbilirubinemia, aminotransferase elevation would predominate, not alkaline phosphatase. (u) C. Gilbert's syndrome presents as unconjugated hyperbilirubinemia so urinary bilirubin would be absent. (u) D. Although this hereditary disorder presents with conjugated hyperbilirubinemia, liver enzymes would not be elevated.

An 80 year-old woman with a 40 pack-year history of smoking presents with progressive dysphagia for solids over the past 6 months. She states that she has often had to regurgitate her food because it "gets stuck." She has had an unintentional weight loss of 20 pounds. Which of the following is the initial diagnostic test of choice? A. Barium esophagography B. Esophageal manometry C. Esophageal motility studies D. Esophageal pH monitoring

(c) A. This patient's history of smoking, unintentional weight loss, and progressive dysphagia for solids makes esophageal neoplasm most likely. The initial less invasive diagnostic test is barium swallow which will detect lumen narrowing (mechanical) lesions such as cancer. (u) B. Manometry measures esophageal pressures which are reduced in GERD or elevated in certain conditions such as achalasia. (u) C. Motility studies are indicated in dysphagia of both solids and liquids looking for a motor dysfunction. (u) D. Esophageal pH monitoring is not indicated in the evaluation of an esophageal malignancy

Ulcerative colitis usually presents with which of the following? A. bloody diarrhea B. toxic megacolon C. fever and left quadrant pain D. alternating constipation and diarrhea

(c) A. Ulcerative colitis typically presents with episodic bloody diarrhea, lower abdominal cramps, and urgency to defecate. (u) B. Toxic megacolon is a complication of ulcerative colitis, but it is not a common presentation. (u) C. Left lower quadrant pain and a palpable mass accompanied by fever is classic for diverticulitis. (u) D. Irritable bowel syndrome usually presents with constipation, painless diarrhea with mucous, or alternating constipation and diarrhea.

An 18 month-old female presents to the Emergency Department having possibly swallowed a hearing aid battery within the past hour. She is drooling and appears anxious but parents have noticed no stridor or dyspnea. She has no history of previous esophageal injury. Physical examination is unremarkable. Chest radiograph reveals a radiopaque round object at the distal esophagus. Which of the following is the most appropriate treatment option? A. Observation for 24 hours B. Esophagoscopy for removal C. Barium swallow D. Bronchoscopy

(h) A. Batteries must be removed as they can induce mucosal injuries in as little as one hour of contact time. (c) B. Esophagoscopy is the procedure of choice for acutely ingested foreign bodies. (u) C. A barium swallow is a diagnostic option but will not provide treatment. (u) D. Bronchoscopy would be the procedure of choice for an airway foreign body, not esophageal.

A 50 year-old male with history of alcohol abuse presents with acute, severe epigastric pain radiating to the back. The patient admits to an episode of coffee ground emesis. On examination he is ill-appearing with a rigid, quiet abdomen and rebound tenderness. Which of the following is the most likely diagnosis? A. Abdominal aortic aneurysm B. Perforated duodenal ulcer C. Acute myocardial infarction D. Cholecystitis

(u) A. A patient with an abdominal aortic aneurysm may present with pain radiating to the back, however would not have coffee ground emesis or an acute abdomen. (c) B. Perforation of a duodenal ulcer causes sudden, severe pain, with rebound tenderness and rigid abdomen on physical examination. It is often associated with coffee ground emesis. (u) C. A patient with an acute myocardial infarction may have pain radiating to the back, however would not have hematemesis or an acute abdomen. (u) D. Cholecystitis presents with right upper quadrant pain and is not typically associated with coffee ground emesis or rebound tenderness.

Which of the following would be consistent for a person who has a successful response to the hepatitis B immunization series? A. HBsAg positive; anti-HBc positive; anti-HBs negative B. HBsAg negative; anti-HBc positive; anti-HBs positive C. HBsAg negative; anti-HBc negative; anti-HBs positive D. HBsAg negative; anti-HBc negative; anti-HBs negative

(u) A. A positive HBsAg and positive anti-HBc with a negative anti-HBs would indicate either acute infection or chronic hepatitis B infection. (u) B. A negative HBsAg with a positive anti-HBc and a positive anti-HBs would indicate a previous infection of hepatitis B. (c) C. A person immunized against hepatitis B would have a positive anti-HBs with negative HBsAg and negative anti-HBc. (u) D. A person with a negative HBsAg, anti-HBc and anti-HBs is nonimmune and has never received the hepatitis B immunization series.

Which of the following clinical findings would be seen in a patient with food poisoning caused by Staphylococcus aureus? A. Ingestion of mayonnaise-based salads 48 hours earlier B. Bloody diarrhea with mucus for one week C. Abdominal cramps and vomiting for 48 hours D. High fever for 1 week

(u) A. A preformed toxin causes staphylococcal food poisoning; it has a short incubation period of 1-8 hours. (u) B. Because Staphylococcus aureus does not invade the mucus, blood and mucus are not seen with this noninflammatory cause of food poisoning. (c) C. Abdominal cramps, nausea, vomiting, and watery diarrhea typically last 1-2 days with staphylococcal food poisoning. (u) D. Staphylococcal food poisoning may be associated with low-grade fever or subnormal temperature.

A patient presents with abdominal pain. On deep palpation of the left lower quadrant the patient notes increased pain in the right lower quadrant. This is noted as a positive A. Psoas sign. B. Murphy's sign. C. Rovsing's sign. D. Obturator sign.

(u) A. A psoas sign is positive when the supine patient notes pain with flexion of the hip against resistance. (u) B. A Murphy's sign is positive when the patient experiences abrupt cessation of deep breathing with palpation of the gallbladder. (c) C. A Rovsing's sign is positive when the patient experiences right lower quadrant pain with deep palpation of the left lower quadrant. (u) D. The obturator sign is positive when the supine patient experiences pain when the right leg is flexed at the hip and knee and the hip is externally and internally rotated.

A 40 year-old male presents with several months of abdominal pain and nausea. Endoscopy reveals an irregular 1 x 2 cm area of loss of rugal folds near the antrum. Biopsies show extensive mucosal and submucosal infiltration by Blymphocytes. The patient is given appropriate treatment and the lesion resolves. Which infectious agent is most likely to be associated with these findings? A. Aspergillus fumigatus B. Helicobacter pylori C. Human papilloma virus D. Salmonella typhi

(u) A. A. fumigatus would be associated with pulmonary, not gastrointestinal disease. (c) B. H. Pylori causes gastric mucosal inflammation with PMN's and lymphocytes. Infection causes nausea and abdominal pain. Inflammation may be confined to the superficial gastric epithelium or may extend deeper resulting in varying degrees of gland atrophy. Eradication of H. Pylori with appropriate therapy leads to resolution of the chronic gastritis. (u) C. Human papilloma virus is primarily a urogenital disorder. (u) D. Salmonella typhi infection would present with fevers, malaise, vomiting, and other symptoms. This diagnosis is best made with blood cultures.

A 65 year-old patient with adenocarcinoma of the colon in remission presents for follow-up. Which of the following tumor markers should be monitored? A. AFP B. CEA C. CA 19-9 D. CA-125

(u) A. AFP is used to monitor recurrence of hepatocellular carcinoma. (c) B. CEA is used to monitor recurrence of colon carcinoma. (u) C. CA 19-9 is used to monitor recurrence of pancreatic carcinoma. (u) D. CA-125 is used to monitor recurrence of ovarian carcinoma.

A patient presents with nausea, vomiting, and fever for one day. She describes periumbilical abdominal pain which has settled in her right lower quadrant. On examination she has right lower quadrant pain with deep left lower quadrant palpation. This finding is known as which of the following? A. Psoas sign B. Rebound tenderness C. Rovsing's sign D. Obturator sign

(u) A. Abdominal pain with flexion of the right hip against pressure is a psoas sign. (u) B. Right lower quadrant pain with quick withdrawal of the right lower quadrant palpation is rebound tenderness. (c) C. Right lower quadrant pain with deep left lower quadrant palpation is Rovsing's sign. (u) D. Right lower quadrant pain with internal rotation of the right hip while both the hip and knee are flexed is obturator sign.

A 16 day-old male presents in the office with a history of vomiting after feeding for the past 2 days. The vomiting has become progressively worse and the mother describes it as very forceful, sometimes hitting the floor 6 feet away. She says the neonate is always hungry. On physical examination, it is noted that he is 2 ounces below birth weight, and has a small palpable mass (about 1.5 cm) in the epigastrium. The most likely diagnosis is A. achalasia. B. tracheoesophageal fistula. C. pyloric stenosis. D. Meckel's diverticulum.

(u) A. Achalasia is uncommon under the age of 5 and the child presents with retrosternal pain and dysphagia. (u) B. Tracheoesophageal fistula presents with increased secretions, choking, cyanosis, and respiratory distress within the first few hours of life. (c) C. Pyloric stenosis begins between 2 to 4 weeks of age with vomiting that becomes projectile after each feeding. An olive-size mass can often be felt in the epigastrium. (u) D. Meckel's diverticulum presents with painless rectal bleeding. Vomiting is rare unless obstruction has occurred.

A 72 year-old male presents to the ED complaining of acute onset of severe diffuse abdominal pain of four hours duration. He states that he has vomited twice since the onset of pain. He also complains of three days of constipation. He is afebrile and the physical examination is noteworthy for a distended, diffusely tender abdomen with normoactive bowel sounds. His rectal exam reveals hemoccult positive brown stool.Medications include omeprazole (Prilosec) for GERD, digoxin and warfarin (Coumadin) for atrial fibrillation,OTC multivitamins and stool softeners. The abdominal and chest x-rays show no abnormalities. Which of the following is the most likely diagnosis? A. Acute cholecystitis B. Mesenteric infarction C. Perforated duodenal ulcer D. Small bowel obstruction

(u) A. Acute cholecystitis causes right upper quadrant pain with radiation into the shoulder and does not cause hemoccult positive stools. (c) B. Acute onset of severe diffuse abdominal pain in a person with atrial fibrillation warrants the suspicion of mesenteric infarction. Vomiting and constipation may be seen, along with occult blood in the stool. Bowel sounds may be normal. (u) C. Abdominal and chest x-rays would reveal free air under the diaphragm in a ruptured duodenal ulcer and asmall bowel obstruction would reveal air-fluid levels with distended bowel loops. (u) D. See C for explanation.

A 25 year-old man presents with odynophagia and dysphagia. On endoscopic examination, small, white, patches with surrounding erythema are noted. Silver stain is positive for hyphae. The best treatment option for this patient is A. acyclovir (Zovirax). B. omeprazole (Prilosec). C. fluconazole (Diflucan). D. penicillin G.

(u) A. Acyclovir is an antiviral used in the treatment of herpes esophagitis. (u) B. Omeprazole is a proton pump inhibitor used in the treatment of gastroesophageal reflux disease with esophageal ulceration and peptic ulcer disease and is not indicated in the treatment of infectious esophagitis. (c) C. The patient has Candida esophagitis and the treatment of choice is fluconazole. (u) D. Penicillin G is an antibiotic and is not effective against fungal infections.

A 25 year-old man presents with odynophagia and dysphagia. On endoscopic examination, small, white, patches with surrounding erythema of the esophagus are noted. Silver stain is positive for hyphae. The best treatment option for this patient is A. acyclovir (Zovirax). B. omeprazole (Prilosec). C. fluconazole (Diflucan). D. penicillin G.

(u) A. Acyclovir is an antiviral used in the treatment of herpes esophagitis. (u) B. Omeprazole is a proton pump inhibitor used in the treatment of gastroesophageal reflux disease with esophageal ulceration and peptic ulcer disease and is not indicated in the treatment of infectious esophagitis. (c) C. The patient has Candida esophagitis and the treatment of choice is fluconazole. (u) D. Penicillin G is an antibiotic and is not effective against fungal infections.

Which of the following presents the greatest risk factor for the development of pancreatic cancer? A. Alcohol abuse B. Coffee consumption C. Cigarette smoking D. Lean body mass

(u) A. Alcohol abuse, coffee consumption and a lean body mass are not proven risk factors for pancreatic cancer. (u) B. See A for explanation. (c) C. Cigarette smoking is the most consistent risk factor for the development of pancreatic cancer. (u) D. See A for explanation.

A male patient presents for a routine physical examination. He denies chronic health problems, regular medication use or previous surgeries. He exercises 4-5 times weekly, does not use tobacco products, and consumes alcohol in moderation. His last intake of alcohol was two weeks ago while on a trip to Mexico. His review of systems and physical examination are both negative. Routine labs were drawn which were significant for the following: Total serum bilirubin 3.5 mg/dL (0.3-1.0 mg/dL), direct bilirubin 0.2 mg/dL (0.1-0.3 mg/dL), AST 35 U/L (0-35 U/L), ALT 30 U/L (0-35 U/L), Alkaline Phosphatase 48 U/L (30-120 U/L) GGT 12 U/L (1-94 U/L). What is the most likely diagnosis in this patient? A. Alcoholic hepatitis B. Crigler-Najjar syndrome C. Gilbert's syndrome D. Wilson's disease

(u) A. Alcoholic hepatitis presents with elevated ALT and AST. (u) B. The Crigler-Najjar syndrome is typified by an isolated elevation in the unconjugated serum bilirubin and usually causes the bilirubin to be elevated at higher levels (6-45 mg/dL). (c) C. Gilbert's syndrome is a relatively common cause of mild isolated elevations in indirect serum bilirubin. (u) D. Wilson's disease is a disorder of impaired biliary copper excretion which among many other problems will cause a rise in total serum bilirubin but also significant elevations in AST.

Which of the following is the most effective prophylaxis for traveler's diarrhea? A. Metronidazole B. Ciprofloxacin C. Tetracycline D. Ampicillin

(u) A. Ampicillin, tetracycline, and metronidazole do not cover the common causes of traveler's diarrhea. (c) B. Options for prophylaxis of traveler's diarrhea include norfloxacin, ciprofloxacin, ofloxacin, or trimethoprim/sulfamethoxazole. (u) C. See A for explanation. (u) D. See A for explanation.

A patient presents with a long history of dyspepsia refractory to maximum appropriate therapy. A recent upper GI series revealed multiple gastric and duodenal ulcerations with prominent mucosal folds. What diagnostic study would be confirmatory of this patient's suspected diagnosis? A. EGD with duodenal biopsy B. Serum gastrin level C. Helicobacter pylori serology D. Abdominal CT scan

(u) A. An endoscopic biopsy of the duodenum may be necessary in some cases to ensure the absence of cancer, etc., but a biopsy in this area is not confirmatory for Zollinger-Ellison Syndrome, which is highly suspect, nor does it allow samples of the gastric mucosa to be studied. (c) B. Serum gastrin levels can be elevated for many reasons; however significant elevations in a fasting state as well as with the secretin stimulation test are confirmatory for Zollinger-Ellison syndrome which is highly suspect in this case. (u) C. Helicobacter pylori can cause symptoms refractory to medical management and an infection can occur concomitantly with Zollinger-Ellison syndrome, but it does not confirm the diagnosis of a gastrinoma or similar hypersecretory disease. (u) D. An abdominal CT scan is not specific enough to confirm the diagnosis of Zollinger-Ellison syndrome.

A 36 year-old truck driver presents with a very tender mass that has developed between his buttocks over the past 4days. Examination shows a small pit-like opening in the midline near the coccyx which is surrounded by an erythematous halo. The area is exquisitely tender and pressure produces purulent drainage from the opening. Which of the following is the most likely diagnosis? A. Anorectal fistula B. Hidradenitis suppurativa C. Perirectal abscess D. Pilonidal cyst

(u) A. Anorectal fistula is associated with perirectal abscess if there is a persistent communication with the crypt after draining. It may create a fistula from the anus to the perirectal skin. (u) B. Hidradenitis is a severe, chronic disabling skin infection of the apocrine sweat glands that can occur in the axilla and groin or perineum area and usually presents with multiple abscesses or sinus tracts. (u) C. Cryptoglandular abscess typically presents with continuous throbbing pain, which is worse with walking. Examination will reveal a tender perianal or rectal mass. Signs of sepsis are possible. (c) D. Pilonidal cyst presents with a tender draining abscess or chronic sinus over the sacrococcygeal or perianal region. It is thought to be due to distended and obstructed hair follicles and rupture into subcutaneous tissues with inspissated hair. On exam the patient will have midline pits near the coccyx or sacrum.

Which of the following laboratory tests, if positive, would be most indicative of Crohn's disease? A. Antineutrophil cytoplasmic antibodies (ANCA) B. Antiendomysial antibodies (AEA) C. Antinuclear antibodies (ANA) D. Anti-Saccharomyces cerevisiae antibodies (ASCA)

(u) A. Antineutrophil cytoplasmic antibodies is positive in 60-70% of patients with ulcerative colitis while only 5-10% of Crohn's disease patients have positive results. (u) B. The antiendomysial IgA antibodies are directed against tissue transglutaminase and has a 90-95% specificity for celiac sprue. (u) C. Antinuclear antibodies are elevated in a wide variety of autoimmune disorders and therefore are a nonspecific finding. (c) D. About 60-70% of patients with Crohn's disease are positive for these antibodies which are directed at the cell walls of S. cerevisiae while only 10-15% of ulcerative colitis is found to be positive.

A patient with a history of severe peptic ulcer disease is 5 weeks status post Billroth I surgery. One week ago he restarted his normal diet and has had the onset of severe nausea, abdominal cramping, and light-headedness that occur approximately thirty minutes after eating. The abdominal exam reveals a healing surgical scar without areas of unusual tenderness or any palpable masses. Which of the following is the most likely diagnosis? A. Anxiety disorder B. Celiac sprue C. Dumping syndrome D. Irritable bowel syndrome

(u) A. Anxiety disorders can cause a wide variety of somatic syndromes such as those mentioned, but generally not with such a straightforward dietary trigger. (u) B. Celiac sprue can cause similar symptoms as those listed, can develop at any age and can be worsened by the ingestion of gluten containing products. The patient's recent surgery makes dumping syndrome a much greater possibility. (c) C. Dumping syndrome typically occurs after Billroth type I surgeries as well as gastric bypass surgeries when the patient attempts to eat a large amount of simple sugars. (u) D. Irritable bowel syndrome is a diagnosis of exclusion and is associated with alternation in bowel habits.

Which of the following is suggestive of thiamine deficiency? A. Ataxia B. Bleeding C. Cheilosis D. Diarrhea

(u) A. Ataxia, mental deficits, horizontal nystagmus, muscle weakness and atrophy, and cardiomegaly are all clinical findings in thiamine deficiency. (u) B. A deficiency of Vitamin K would result in bleeding and an elevated prothrombin time. (c) C. Cheilosis, mucocutaneous lesions at the corners of the mouth, is the result of riboflavin deficiency. (u) D. A deficiency of niacin will result in diarrhea, dementia, and dermatitis.

A 3 week-old infant is evaluated for persistent projectile vomiting described as breast milk without bile or blood. The abdomen is distended before vomiting and a small, mid-epigastric mass is palpable after vomiting. Which of the following is the most appropriate diagnostic study for the evaluation of this patient? A. Barium enema B. Esophageal manometry C. H. pylori stool antigen D. Upper GI contrast radiographs

(u) A. Barium enema is usually both diagnostic and therapeutic for intussusception which usually presents as recurring paroxysms of abdominal pain, vomiting and bloody diarrhea. (u) B. Esophageal manometry is used to measure LES pressure which may be increased in esophageal achalasia. Usually a child over 5 years presents with dysphagia, retrosternal pain and slow eating. (u) C. Positive H. pylori stool antigen is used to test for bacterial infection associated with PUD that presents with pain and bleeding. (c) D. This infant has pyloric stenosis and an upper GI series will reveal a narrowed distal stomach with double tract of barium.

A 66 year-old female presents to your office complaining of progressive difficulty swallowing over the last 6 months. Initially she had difficulty only with meats, but now she has dysphagia with other foods as well. Which of the following is the most appropriate initial diagnostic study? A. Barium swallow B. Endoscopy C. CT scan D. Urea breath test

(u) A. Barium swallow may be used in the evaluation of dysphagia, but does not allow for biopsy to be performed. (c) B. Endoscopy is the diagnostic study of choice in a patient with progressive dysphagia because of its ability to obtain tissue for diagnosis. (u) C. See B for explanation. (u) D. See B for explanation.

Which of the following tumor markers is useful in monitoring a patient for recurrence of colorectal cancer after surgical resection? A. CA-125 B. Carcinoembryonic antigen C. 5-hydroxindoleacetic acid D. Alpha-1-fetoprotein

(u) A. CA-125 tumor marker is useful as a tumor marker in ovarian cancer (c) B. Carcinoembryonic antigen can be used to monitor a patient for the return of colorectal cancer after treatment. (u) C. 5-hydroxindoleacetic acid is used as a tumor marker in carcinoid syndrome. (u) D. Hepatocellular and testicular cancer can be followed by the use of alpha-1-fetoprotein as a tumor marker.

A 56 year-old diabetic male with a three-month history of epigastric pain after eating says he gets full really fast now. His test for Helicobacter pylori is negative. Which of the following is the recommended medication to treat this patient? A. Calcium carbonate (Tums) B. Ranitidine (Zantac) C. Omeprazole (Prilosec) D. Metoclopramide (Reglan)

(u) A. Calcium carbonate has no effect on gastric emptying. (u) B. H2 blockers have no effect on gastric emptying. (u) C. Prilosec has no effect on gastric emptying. (c) D. Metoclopramide accelerates gastric emptying and is the treatment of choice for this patient.

A person presenting with bleeding esophageal varicies should be treated with which of the following while awaiting arrival of endoscopy? A. Carafate (Sucralfate) B. Octreotide (Sandostatin) C. Omeprazole (Prilosec) D. Enoxaparin (Lovenox)

(u) A. Carafate is not indicated in acute gastrointestinal bleeding. (c) B. Octreotide is a vasoacctive drug used in the treatment of GI bleeding as well as somatostatin, vasopressin, and terlipressin. Somatostatin and octreotide are preferred due to safety and less incidence of serious side effects. (u) C. Omeprazole, a proton pump inhibitor, is not indicated in acute gastrointestinal hemorrhage. (h) D. Enoxaparin will increase bleeding and therefore contraindicated in GI bleeding.

A 45 year-old male presents with a long history of ulcerative colitis and recent progressive complaints of right upper quadrant pain, weight loss, fever and most recently, a rapid onset of jaundice with pruritus. Labs revealed elevated bilirubin and alkaline phosphatase. Viral serologies were negative. An endoscopic cholangiogram showed areas of stenosis and dilation throughout the bile duct system. What is the most likely diagnosis? A. Choledocholithiasis B. Hepatic carcinoma C. Portal hypertension D. Primary sclerosing cholangitis

(u) A. Choledocholithiasis can most certainly cause elevated bilirubin and other liver function tests when obstruction occurs. It also occurs more often in persons with sclerosed or narrowed bile ducts however it does not generally carry a poor prognosis and can be generally recognized by its typical symptoms. (u) B. Hepatic carcinoma does not cause areas of varied stenosis and dilation in the biliary tree. (u) C. Portal hypertension may present with jaundice but generally has significant ascites as well that helps to distinguish this disorder. It also does not result in the bile duct pattern mentioned. (c) D. Primary sclerosing cholangitis (PSC) results in diffuse intra- and extrahepatic duct sclerosing with dilatation proximal to these areas.

A 65 year-old homeless male with a history of pancreatitis is seen in the emergency department for vomiting, upper abdominal pain, back pain and weakness. He is cachetic, pale and jaundiced. A 4-5 cm mass is palpable in the mid to right hypochondrium. What is the most likely diagnosis? A. Chronic cholecystitis B. Carcinoma of head of pancreas C. Fibrolipoma D. Primary biliary cirrhosis

(u) A. Chronic cholecystitis is not typically associated with weight loss or cachexia. There would not be a palpable mass. (c) B. Seventy-five percent of pancreatic cancers are in the head. Risk factors include age, tobacco use, obesity, chronic pancreatitis, family history and previous abdominal radiation. (u) C. Fibrolipoma may present as an abdominal mass, but would not cause weight loss and illness. (u) D. Primary biliary cirrhosis most commonly presents with generalized urticaria and is not associated with an abdominal mass.

A 19 year-old female presents with intermittent episodes of loose stools over the past 6 to 8 months. She admits to crampy abdominal pain and bloating that seems to be more associated with her stress level than with food intake. She denies fever, melena, or nocturnal symptoms. Examination reveals normoactive bowel sounds and mild lower abdominal tenderness on palpation without rebound tenderness or organomegaly. Which of the following is the treatment of choice in this patient? A. Cimetidine (Tagamet) B. Metronidazole (Flagyl) C. Metoclopramide (Reglan) D. Dicyclomine (Bentyl)

(u) A. Cimetidine (Tagamet) is used in the management of GERD and PUD not IBS. (u) B. Metronidazole (Flagyl) is indicated in the treatment of Clostridium difficile associated diarrhea, not irritable bowel syndrome. (u) C. Metoclopramide (Reglan) is used in the management of nausea and vomiting, not irritable bowel syndrome. (c) D. Dicyclomine (Bentyl) and other anticholinergic agents can help relieve abdominal symptoms of cramping and bloating associated with irritable bowel syndrome.

Which of the following medications used in the treatment of peptic ulcer disease is classified as a proton pump inhibitor? A. Cimetidine (Tagamet) B. Sucralfate (Carafate) C. Omeprazole (Prilosec) D. Misoprostol (Cytotec)

(u) A. Cimetidine is an H2 receptor antagonist. (u) B. Sucralfate (Carafate) is a cytoprotective agent. (c) C. Omeprazole is a proton pump inhibitor. (u) D. Misoprostol is a prostaglandin analog.

A 33 year-old patient returns from a community picnic that was held on a hot July day. The patient developed severe explosive diarrhea, nausea, vomiting and abdominal cramps six hours after having homemade potato salad and hot dogs. Multiple people who ate at this picnic have similar symptoms. There is no associated fever. What is the most likely causative organism for these symptoms? A. Clostridium perfringens B. Campylobacter jejuni C. Shigella D. Staphylococcus aureus

(u) A. Clostridium perfringens has a 12 to 24 hour incubation period and symptoms are limited to diarrhea and crampy abdominal pain but nausea and vomiting are unusual. (u) B. Campylobacter jejuni, the most common cause of acute bacterial diarrhea, has a prodrome period of 12 to 24-hours with headache, fever, and malaise that is followed by crampy abdominal pain and diarrhea. (u) C. Shigella infection causes bacterial dysentery in tropical regions most commonly and transmission is usually from person to person rather than from food-borne spread. Nausea and vomiting are unusual. (c) D. Staphylococcus aureus is an acute illness caused by the ingestion of enterotoxin-producing organism. It is the only common acute ingestion of a preformed toxin. The clinical picture in the question is the classic presentation for this illness. Foods most responsible for this infection are ham, cream, custard pastries, and mayonnaise-based salads.

What nail finding is most consistent with cirrhotic liver disease? A. Clubbing B. Pitting C. Terry's nails D. Beau's lines

(u) A. Clubbing is a bulbous swelling of soft tissue at the nail base most commonly seen in congestive heart disease, interstitial lung disease, lung cancer, irritable bowel syndrome, and malignancies. (u) B. Pitting appears as punctuate depressions of the nail caused by defective layering of the superficial nail plate by the proximal nail matrix. Pitting is usually associated with psoriasis. (c) C. Terry's nails occur when the nail plate turns white with a ground glass appearance, a distal band of reddish brown, and obliteration of the lunula. Terry's nails are seen in liver disease, usually cirrhosis. (u) D. Beau's lines are transverse depressions of the nails and are usually bilateral and result from temporary disruption of proximal nail growth from systemic illness.

Which of the following is considered to be the treatment of choice for the pruritus that occurs with primary biliary cirrhosis? A. Colchicine (Colzalide) B. Atorvastatin (Lipitor) C. Cholestyramine (Questran) D. Enalapril (Vasotec)

(u) A. Colchicine is used to improve the biochemical abnormalities which may slow the progression of the disease. (u) B. Statins have a role in the management of hyperlipidemia but are not effective in this disease. (c) C. Cholestyramine, a bile salt sequestrant, is able to decrease the pruritus that occurs from the bile stasis and granulomas. (u) D. ACE inhibitors are effective as antihypertensives and in preserving renal function in those with proteinuria, but they have no role in the management of primary biliary cirrhosis.

Which of the following medications is most useful in maintaining remission in a patient with ulcerative colitis? A. Oral corticosteroids B. Corticosteroid enemas C. Sulfasalazine D. Macrodantin

(u) A. Corticosteroids, given orally or via enema preparations, are used for acute flare-up of ulcerative colitis but are not effective when given as prophylactic agents. (u) B. See A for explanation. (c) C. Sulfasalazine, olsalazine, and mesalamine are effective in maintaining remission in patients with ulcerative colitis. (u) D. Long-term antibiotic therapy with Macrodantin is used as a prophylactic agent to prevent urinary tract infections and has no role in the treatment or prevention of ulcerative colitis.

Which of the following is the diagnostic study of choice in the evaluation of Zollinger-Ellison syndrome? A. D-xylose absorption test B. Gastrin level C. 5-hydroxyindoleacetic acid assay (5-HIAA) D. Urea breath test

(u) A. D-xylose absorption test is used in the evaluation of possible gluten-induced enteropathy. (c) B. Zollinger-Ellison Syndrome (ZES) is characterized by elevated levels of serum gastrin resulting from gastrinsecreting tumors usually located in the duodenum or pancreas. (u) C. 5-HIAA levels are elevated in metastatic carcinoid tumors. (u) D. The urea breath test is used to evaluate ulcers caused by H. pylori infection.

A 62 year-old patient with arthritis is on daily naproxen therapy. Which of following medications is used to protect against GI complications? A. diphenoxylate (Lomotil) B. sulfasalazine (Azulfidine) C. metoclopramide (Reglan) D. misoprostol (Cytotec)

(u) A. Diphenoxylate is used in the treatment of diarrhea. (u) B. Sulfasalazine is used in the treatment of inflammatory bowel disease. (u) C. Metaclopramide is used in the treatment of nausea and vomiting. (c) D. Misoprostol is an analog of prostaglandins and used in the treatment of NSAID induced ulcer disease.

The parents of a 16 year-old male presents to the clinic with their son asking that you examine him. Over the past 9-12 months he has developed behavioral problems and emotional lability. Physical examination reveals a well-developed male who is cooperative with exam but tends to be easily distracted. It is noteworthy for dysarthria, a resting tremor and the presence of gray-green pigmentation surrounding each pupil. The most likely diagnosis is A. drug abuse. B. hemochromatosis. C. Wilson's disease. D. Parkinsonism.

(u) A. Drug abuse would account for the personality changes and possibly the resting tremor. It would not account for the ocular findings. (u) B. Hemochromatosis has its onset usually after the age of 50 years and would not produce neuropsychiatric symptoms, the tremor or the ocular abnormalities. (c) C. Wilson's disease results in the excessive deposition of copper in the liver and brain. Kayser-Fleisher rings are the result of granular deposits in the eye and are pathognomonic for Wilson's disease. (u) D. Parkinsonism would not result in the ocular findings and this would be a very unusual age of onset for Parkinsonism.

The drug of choice in an adult for empirical treatment of an ill-appearing patient with infectious diarrhea who has recently returned from Mexico is A. amoxicillin. B. ceftriaxone (Rocephin). C. ciprofloxacin (Cipro). D. doxycycline.

(u) A. Due to increasing antibiotic resistance, amoxicillin is no longer recommended in the treatment of infectious diarrhea. (u) B. See C for explanation. (c) C. Fluoroquinolones like Ciprofloxacin are the class of drugs used in the empirical treatment of infectious diarrhea. Alternatives include trimethoprim/sulfamethoxazole or erythromycin. (u) D. See C for explanation.

Which of the following diagnostic studies is indicated in the evaluation of an upper gastrointestinal bleed? A. Esophageal manometry B. Bleeding scan C. Upper endoscopy D. Barium swallow

(u) A. Esophageal manometry is not indicated in the evaluation of upper gastrointestinal bleeding. (u) B. Bleeding scans are most useful to evaluate occult GI bleeding. Radioactive isotope tracing is useful to concentrate bleeding sites at a single location. It is most useful to find bleeding that is occurring in the large or small bowel rather than in the upper GI sites. (c) C. Endoscopy is the evaluation modality of choice in patients with upper GI bleeding. The advantage of this technique is that it can be used for both diagnostic and therapeutic purposes. (u) D. Barium swallow is not indicated in the evaluation of upper gastrointestinal bleeding.

Which of the following is a common physical examination finding in early intestinal obstruction? A. high fever. B. profuse flatulence. C. rebound tenderness D. hyperactive, high-pitched bowel sounds

(u) A. Fever and rebound tenderness occur only if a perforation has occurred. (u) B. Profuse flatulence is not noted in early intestinal obstruction. (u) C. See A for explanation. (c) D. Abdominal distention and high-pitched, hyperactive bowel sounds are common in early intestinal obstruction.

The initial manifestation of acute pancreatitis is often A. generalized pruritus. B. epigastric pain. C. epigastric mass. D. anorexia.

(u) A. Pruritus presents as a symptom only when acute pancreatitis occurs as a result of obstruction of the distal common bile duct. (c) B. The typical findings in acute pancreatitis include nausea, vomiting, and abdominal pain. Epigastric pain, generally abrupt in onset, is steady and severe. (u) C. Epigastric mass is more common in pancreatic cancer. (u) D. Anorexia is a common finding with chronic pancreatitis, but uncommon with acute pancreatitis.

A 3 week-old male infant presents with recurrent regurgitation after feeding that has progressed to projectile vomiting in the last few days. The mother states that the child appears hungry all of the time. She denies any diarrhea in the child. Which of the following clinical findings is most likely? A. Bile-stained vomitus B. Hemoccult positive stools C. Olive-sized mass in the right upper abdomen D. Sausage-shaped mass in the upper-mid abdomen

(u) A. Gastric obstruction, such as that seen with pyloric stenosis, causes vomiting that is not bilious. (u) B. Blood-streaked vomitus, but not hemoccult positive stools, may be seen in pyloric stenosis. (c) C. An olive-sized mass may be palpated in the right upper abdomen in pyloric stenosis and if found, is pathognomonic for pyloric stenosis. (u) D. A sausage-shaped mass may be noted in intussusception, not pyloric stenosis.

Which medication is considered the mainstay of therapy for mild to moderate inflammatory bowel disease? A. Prednisone B. Sulfasalazine C. Metronidazole D. Azathioprine (Imuran)

(u) A. Glucocorticoids are used in moderate to severe inflammatory bowel disease. (c) B. Sulfasalazine and other 5-aminosalicylic acid drugs are the cornerstone of therapy in mild to moderate inflammatory bowel disease as they have both anti-inflammatory and antibacterial properties. (u) C. Metronidazole may have a role in Crohn's disease after ileal resection but has no role in the treatment of mild to moderate disease. (u) D. Azathioprine is used in severe, glucocorticoid-dependent inflammatory bowel disease.

Which of the following complications of Crohn's disease most frequently requires surgery? A. Hemorrhage B. Intra-abdominal abscess C. Malabsorption D. Perforation

(u) A. Hemorrhage and perforation are uncommon in Crohn's disease. (c) B. Surgery is the treatment of choice for an abscess, which is a common complication of Crohn's disease. (u) C. Malabsorption is often a complication of the surgery itself. (u) D. See A for explanation.

A pregnant female presents at 32 weeks gestation with painless rectal bleeding and a bulging perianal mass when straining which goes away. Which of the following is the most appropriate management of this patient? A. Hemorrhoidectomy B. Metronidazole (Flagyl) C. Psyllium (Metamucil) D. Sclerotherapy

(u) A. Hemorrhoidectomy and sclerotherapy are reserved for severe Grade III and IV hemorrhoids. (u) B. Metronidazole is not indicated in the treatment of hemorrhoids. (c) C. Dietary fiber or psyllium bulk laxatives can be used to decrease straining with defecation. (u) D. See A for explanation.

A patient presents complaining of vague anal discomfort. On examination, the patient is noted to have a few small external hemorrhoids and edema in the anal region. Which of the following is the most appropriate intervention? A. proctoscopy followed by a hemorrhoidectomy B. increased dietary fiber and sitz baths C. hemorrhoidal banding D. inject a sclerosing agent

(u) A. Hemorrhoidectomy should be used for permanently prolapsed internal hemorrhoids. (c) B. Most hemorrhoids respond well to conservative treatment such as fiber and sitz baths. (u) C. Banding and injection of sclerosing agents are used if mild prolapse, enlargement, or intermittent bleeding is present. (u) D. See C for explanation.

Which of the following subtypes of viral hepatitis requires the presence of the hepatitis B virus for replication? A. hepatitis A B. hepatitis C C. hepatitis D D. hepatitis E

(u) A. Hepatitis A is transmitted almost exclusively by the fecal-oral route and does not require the presence of the hepatitis B virus for replication. (u) B. While hepatitis C may occur in IV drug users, it does not require the presence of the hepatitis B virus for replication. (c) C. In the United States, hepatitis D is most commonly seen in persons exposed frequently to blood or blood products, such as drug addicts. It requires the presence of hepatitis B virus for its replication and expression. (u) D. Hepatitis E resembles hepatitis A in its enteric mode of transmission and does not require the presence of the hepatitis B virus for replication.

Which of the following is best described as a hard nodule in the periumbilical region? A. Hodgkin's lymphoma B. Gastric bezoar C. Sister Mary Joseph nodule D. Virchow's node

(u) A. Hodgkin's lymphoma will more likely affect intra-abdominal lymph nodes. (u) B. Gastric bezoar can present as a palpable abdominal mass, but is not indicative of intra-abdominal/intra-pelvic cancer. (c) C. The Sister Mary Joseph nodule can occur with metastatic gastric and pancreatic cancers. The nodule is generally minimally to non-tender to palpation. (u) D. Virchow's node is found in cases of metastatic disease, but is represented by a left supraclavicular lymph node.

A 3 year-old presents with profuse watery diarrhea for the past three days. The child vomited twice yesterday, but not today. On examination, the child is febrile, with pulse of 142, respiratory rate of 18, and blood pressure of 60/40 mmHg. The child is alert and responsive, with no focal findings. Which of the following is the most appropriate intervention? A. Antibiotic therapy B. Loperamide (Imodium) C. 3% normal saline IV infusion D. Oral rehydration

(u) A. In the US, infectious gastroenteritis is most frequently due to a virus. Antibiotic therapy may be second-line in cases where the causative organism is bacterial, is identified, and symptoms continue. (u) B. Loperamide may lead to toxic megacolon. (h) C. 3% normal saline infusion may cause hypernatremia and central pontine myelinosis. (c) D. The goal of therapy for a child with severe gastroenteritis and dehydration is to restore fluid loss. Oral rehydration with an appropriate electrolyte solution is the best option if the child is not actively vomiting and is alert enough to take oral fluids.

A 3 year-old presents with profuse watery diarrhea for the past three days. The child vomited twice yesterday, but not today. On exam, the child is febrile, with pulse of 142, respiratory rate of 18, and blood pressure of 60/40 mmHg. On exam, the child is alert and responsive, with no focal findings. Which of the following is the most appropriate intervention? A. Antibiotic therapy B. Begin soft diet C. IV fluids D. Oral rehydration

(u) A. In the US, infectious gastroenteritis is most frequently due to a virus. Antibiotic therapy may be second-line in cases where the causative organism is bacterial, is identified, and symptoms continue. (u) B. See D for explanation. (a) C. See D for explanation. (c) D. The goal of therapy for a child with severe gastroenteritis and dehydration is to restore fluid loss. Oral rehydration with an appropriate electrolyte solution is the best option if the child is not actively vomiting and is alert enough to take oral fluids. IV fluids should be reserved for those who are unable to take fluids orally.

A newborn weighs 8 pounds at birth. Following established growth indices, what would you expect the infant to weigh at 1 year of age? A. 16 pounds B. 24 pounds C. 32 pounds D. 40 pounds

(u) A. See B for explanation. (c) B. Most infants should be expected to triple their birth weight by 1 year of age. (u) C. See B for explanation. (u) D. See B for explanation

A 72 year-old man presents with acute left lower quadrant abdominal pain. He has nausea, vomiting, and constipation. He has a fever of 101° F and guarding and rebound tenderness in his left lower quadrant. His white blood cell count is elevated. He has no prior history of gastrointestinal disease. Which of the following is the most likely diagnosis? A. Inflammatory bowel disease B. Irritable bowel syndrome C. Viral gastroenteritis D. Acute diverticulitis

(u) A. Inflammatory bowel disease typically presents in a younger population. (u) B. Irritable bowel syndrome is not associated with nausea, vomiting and fever. It usually presents in a younger population. (u) C. Viral gastroenteritis typically does not localize to the left lower quadrant. (c) D. Acute abdominal pain, fever, left lower abdominal tenderness, and leukocytosis are hallmark signs of acute diverticulitis.

A 14 month-old male who attends day care presents with a two-day history of frequent watery stools. His mother states that he had a fever and vomiting the day before but these have resolved. His mother denies pain in the child. The child is mildly dehydrated but otherwise appears well. Stool samples are free of blood and white blood cells. The lab reports no ova or parasites noted in the stool samples. Which of the following is the most likely diagnosis? A. Intussusception B. Viral gastroenteritis C. Shigella D. Lactase insufficiency

(u) A. Intussusception may cause diarrhea, however, after two days the stool would have blood present. The absence of pain or discomfort would also argue against this diagnosis. (c) B. Rotavirus is the most common cause of gastroenteritis in children and this is frequently passed in the daycare setting. (u) C. Patients with Shigella have dysentery with the passage of bloody stools in a toxic-appearing child (u) D. Lactase insufficiency would not have caused fever and vomiting.

The most common initial presenting symptom of primary biliary cirrhosis is A. jaundice B. palmar erythema C. pruritus D. xanthomas

(u) A. Jaundice, palmar erythema and xanthomas appear late in the disease. (u) B. See A for explanation. (c) C. Pruritus is the most common initial symptom in primary biliary cirrhosis due to the accumulation of bile salts. (u) D. See A for explanation.

A patient presents with a concern about an upcoming trip. He states he is traveling to Mexico and has a question about how to treat diarrhea should it develop. In addition to promoting fluid intake which of the following is recommended? A. kaolin-pectin B. ciprofloxacin C. simethicone D. doxycycline

(u) A. Kaolin-pectin is not indicated in the treatment of traveler's diarrhea. (c) B. Ciprofloxacin is indicated in the treatment of traveler's diarrhea. (u) C. Simethicone is used in the treatment of intestinal gas. (u) D. Doxycycline is not indicated in the treatment of traveler's diarrhea due to increasing resistance.

A 23 year-old female presents with a 24-hour history of watery diarrhea, nausea and vomiting after returning from a cruise. On exam, the vitals reveal HR 120, BP 90/60, and RR 20. The abdomen is soft and non-tender, with hyperactive bowel sounds. Which of the following is the most appropriate management at this time? A. loperamide (Imodium) B. IV hydration C. bismuth sulfate (Pepto bismol) D. ciprofloxacin (Cipro)

(u) A. Loperamide is not indicated for acute infectious gastroenteritis in the vomiting patient. (c) B. This is most likely a self-limiting viral gastroenteritis and is treated with hydration and supportive care. (u) C. Bismuth sulfate is useful for treating diarrhea but is not helpful while the patient is vomiting. (u) D. Antibiotics, like ciprofloxacin, are not indicated for viral infections.

A 45 year-old male presents with abdominal pain and one episode of mild hematemesis, which happened days ago. On physical examination, vital signs are stable and he is in no acute distress. Hemoglobin and hematocrit are unremarkable; endoscopy reveals non-bleeding small superficial ulceration of the duodenal bulb. Rapid urease test is positive. Which of the following is the most appropriate treatment at this time? A. Schedule for a selective vagotomy and antrectomy B. Start an antacid along with omeprazole (Prilosec) C. Schedule elective ulcer excision and start sucralfate (Carafate) D. Start omeprazole (Prilosec) and antibiotic therapy against H. pylori

(u) A. Medical therapy should be initiated prior to any consideration of surgery, which is rarely performed secondary to satisfactory ulcer healing with medical therapy. (u) B. While proton pump inhibitors, such as omeprazole, have excellent results in healing duodenal ulcers, this regimen will not treat the H. pylori infection that is documented by the positive rapid urease test. (u) C. While sucralfate can be utilized as a cytoprotectant agent in treatment of active ulcer disease or in maintenance of healed ulcers, surgery to remove the ulcer is not warranted as initial therapy. (c) D. Treatment goals of H. pylori associated ulcers include eradicating the infection with appropriate antibiotics as well as use of a proton pump inhibitor, such as omeprazole, to promote ulcer healing.

A 48 year-old male with a history of AIDS presents with complaints of odynophagia and dysphagia for 1 week. The patient denies nausea, vomiting, diarrhea, constipation or melena. Endoscopic evaluation reveals diffuse, linear plaques which are yellowish-white in color and adhere to the esophageal mucosa. Which of the following is the treatment of choice for this patient? A. Nystatin (Mycostatin) suspension B. Oral acyclovir (Zovirax) C. Oral fluconazole (Diflucan) D. Inhaled triamcinolone acetonide (Azmacort)

(u) A. Nystatin suspension is indicated in the management of oropharyngeal candidiasis not esophageal. (u) B. Oral acyclovir is indicated in the management of herpes esophagitis. (c) C. Oral fluconazole is the treatment of choice for a patient with candidal esophagitis. (u) D. Inhaled corticosteroid is used in the management of eosinophilic esophagitis.

Which of the following is the therapy of choice for long-term management of esophageal varices in a patient who cannot tolerate beta blocker therapy? A. Octreotide (Sandostatin) B. Sclerotherapy C. Transjugular intrahepatic portosystemic shunt D. Sengstaken-Blakemore tube

(u) A. Octreotide is used to control acute variceal bleeding but does not have a role in long-term management. (c) B. Sclerotherapy is effective in decreasing the risk for rebleeding in a patient with esophageal varices. (u) C. Transjugular intrahepatic portosystemic shunt (TIPS) decreases rebleeding more than sclerotherapy or ligation but is associated with a higher incidence of encephalopathy and has shown no decrease in mortality. TIPS is used in patients who fail endoscopic or pharmacologic therapy. (u) D. The Sengstaken-Blakemore tube is rarely used today in the treatment of acute variceal bleeding. These are not indicated for long-term use.

A 16 year-old male is brought into your office by his girlfriend. She states that "he hasn't been himself lately" and seems to fluctuate from being almost "euphoric" to being depressed and irritable. The patient states that "he is really okay" and that he "just feels a little irritable occasionally." On physical examination his pulse is 120 beats/minute, blood pressure is 180/110 mmHg, he is sweating and his pupils are widely dilated. Which of the following is the most likely diagnosis? A. Opiate abuse B. Acute anxiety attack C. Cocaine intoxication D. Bipolar affective disorder

(u) A. Opiate abuse would be suspected with the presence of euphoria, drowsiness, and constricted pupils. More severe cases present with bradycardia, hypotension, coma, or respiratory arrest. (u) B. While acute anxiety disorder may present with tachycardia, agitation, diaphoresis, and hypertension, this diagnosis should be a diagnosis of exclusion. Failure to diagnose cocaine intoxication could lead to severe consequences. (c) C. Cocaine is a stimulant and presenting clinical manifestations of intoxication include agitation, tachycardia,hypertension, diaphoresis, and dilated pupils. (u) D. While the history may suggest the diagnosis of bipolar affective disorder, the physical exam findings do not support this diagnosis.

A 62 year-old male presents with complaints of dyspepsia, early satiety, and dysphagia. What diagnostic study would be indicated in his initial workup? A. PET CT scan B. Abdominal ultrasound C. Gastric emptying studies D. Upper endoscopy

(u) A. PET CT scan is used primarily to follow the course of an already diagnosed cancer. (u) B. Abdominal ultrasonography is performed only when pancreatic or biliary disease is suspected. (u) C. Gastric emptying studies are valuable only in patients with recurrent vomiting. (c) D. Upper endoscopy is indicated for patients over the age of 55 presenting with new-onset symptoms of dyspepsia in order to evaluate for gastric cancer or other serious organic disease. Upper endoscopy is the diagnostic study of choice to diagnose gastroduodenal ulcers, erosive esophagitis, and upper GI malignancy.

A patient presents complaining of periumbilical pain. Which of the following anatomical sites is this finding associated with? A. Bladder B. Stomach C. Pancreas D. Small bowel

(u) A. Pain from the bladder, uterus, or colon causes hypogastric pain. (u) B. Pain from the stomach, duodenum, or pancreas causes epigastric pain. (u) C. See B for explanation. (c) D. Pain from the small intestine, appendix, or proximal colon causes periumbilical pain.

A 33 year-old male returns from an extended camping trip and complains of 5 weeks of mild diarrhea, mostly after breakfast. The stool is described as bulky, greasy, frothy, and malodorous; it is free of blood or pus. He is afebrile and has upper abdominal discomfort, cramps, distention, and excessive flatus. he has lost several pounds. Which of the following is the most likely diagnosis? A. Amebiasis B. Giardiasis C. Schistosomiasis D. Trypanosomiasis

(u) A. Parasitic dysentery more probably would present as febrile illness with blood and/or mucus in watery diarrhea and not associated with meals. (c) B. This is a classic presentation of diarrheal illness due to Giardia and in its chronic state can cause malabsorption and therefore be associated with foul smelling, oily or greasy stools with weight loss. (u) C. Chronic infection by Schistosoma sp.with intestinal involvement presents as abdominal pain, irregular bowel movements and blood in the stool. The clinical syndrome usually presents with fever and chills andorganomegaly. (u) D. Febrile illness is due to infection by Trypanosoma sp.(sleeping sickness) which may have CVS and CNS clinical features.

Which of the following treatments is used to reduce the hepatic complications of hemochromatosis? A. Penicillamine B. Paracentesis C. Albumin D. Phlebotomy

(u) A. Penicillamine is a chelating agent that may be used in the treatment of Wilson's disease in which there is a deficiency of the copper-binding protein ceruloplasmin resulting in impaired copper excretion into the bile. (u) B. Paracentesis may be performed in patients with tense ascites which can symptomatically improve the patient but does not alter long-term prognosis. (u) C. Patients with long-standing liver failure have decreased albumin levels but this is not a treatment normally performed for hemochromatosis. (c) D. Hemochromatosis is an autosomal recessive inherited disorder that causes cirrhosis, diabetes, and bronze pigmentation to the skin due to the abnormal accumulation of iron in tissues. Intensive phlebotomy is the treatment of choice until the iron overload is corrected.

A 32 year-old male with history of tobacco abuse presents with an intermittent burning sensation in his chest for six months, worsening over the past 2 weeks. His wife has noticed episodes of coughing at night. He denies dysphagia, weight loss, hematemesis, or melena. His vital signs are all normal and physical examination is unremarkable. Which of the following is the most likely diagnosis? A. Peptic ulcer disease B. Acute gastritis C. Gastroesophageal reflux disease D. Esophageal stricture

(u) A. Peptic ulcer disease typically presents with midepigastric pain and is not usually associated with cough. (u) B. Acute gastritis would have a duration of less than 2 weeks. (c) C. Gastroesophageal reflux disease presents with at least weekly episodes of heartburn and typically occurs after meals and upon reclining. Patients may complain of regurgitation, chronic cough, laryngitis, or sore throat. (u) D. Esophageal stricture develops from long term gastroesophageal reflux disease. Patients with esophageal stricture usually have dysphagia.

A 42 year-old male with a history of constipation presents with complaints of severe pain with defecation described as feeling like he is "tearing apart." He has also noted occasional small amounts of blood on toilet paper. External examination of the rectum is unremarkable and an internal rectal exam cannot be performed due to severe pain when attempted. Which of the following is the most likely diagnosis? A. Proctitis B. Anal fissure C. Rectal prolapse D. Internal hemorrhoids

(u) A. Proctitis is usually caused by anorectal infections that produce symptoms of anorectal discomfort, tenesmus, constipation, and discharge from the rectum. (c) B. Anal fissures are easily diagnosed from history alone with the classic finding of severe pain upon defecation.Constipation is also a common cause of the trauma that leads to development of a fissure. (u) C. Rectal prolapse is commonly seen in elderly females with complaints that include an anal mass, rectal bleeding and a change in bowel habits. (u) D. While internal hemorrhoids may cause rectal bleeding, tearing pain is an uncommon complaint unless there is evidence of thrombosis of irreducible tissue.

Which of the following is the most common cause of ascites? A. Nephrotic syndrome B. Portal hypertension C. Bacterial peritonitis D. Uterine malignancy

(u) A. See B for explanation. (c) B. Over 80% of patients with ascites have portal hypertension secondary to chronic liver disease. Infection, malignancy, and nephrotic syndrome are common causes of non-portal hypertensive ascites. (u) C. See B for explanation. (u) D. See B for explanation.

A patient presents with abdominal pain in the right lower quadrant, examination reveals increased pain in the right lower quadrant on deep palpation of the left lower quadrant. This commonly known as which of the following? A. Psoas sign B. Murphy's sign C. Rovsing's sign D. Obturator sign

(u) A. Psoas sign is right lower quadrant pain with right leg extension. (u) B. Murphy's sign is seen in liver and gallbladder disease in which the patient abruptly halts deep inspiration due to discomfort as the examiners hand applies pressure to the right upper quadrant. (c) C. A positive Rovsing's sign can be elicited in a patient with appendicitis when increased pain occurs in the right lower quadrant upon palpation of the left lower quadrant. (u) D. Obturator sign is right lower quadrant pain with internal rotation of the hip.

What is the pathologic mechanism of Hirschsprung's disease? A. Pyloric circular muscle hypertrophy causing gastric outlet obstruction B. Absence of ganglion cells in the mucosal and muscular layers of the colon C. A defect in the diaphragm leading to protrusion of the abdominal viscera into the thoracic cavity D. Absence of relaxation of the lower esophageal sphincter and lack of peristalsis in the esophageal body

(u) A. Pyloric circular muscle hypertrophy causing gastric outlet obstruction describes pyloric stenosis. (c) B. Hirschsprung's disease results from an absence of ganglion cells in the mucosal and muscular layers of the colon. (u) C. A defect in the diaphragm leading to protrusion of the abdominal viscera into the thoracic cavity describes congenital diaphragmatic hernia. (u) D. Absence of relaxation of the lower esophageal sphincter and lack of peristalsis in the esophageal body describes achalasia of the esophagus.

Which of the following medications decreases the exchange of hydrogen for potassium by inhibiting hydrogen, potassium-ATPase? A. Ranitidine (Zantac) B. Misoprostol (Cytotec) C. Sucralfate (Carafate) D. Omeprazole (Prilosec)

(u) A. Ranitidine is an H2 receptor antagonist. (u) B. Misoprostol is a prostaglandin analogue. (u) C. Sucralfate is a coating agent. (c) D. Omeprazole, a proton pump inhibitor, works by inhibiting hydrogen, potassium-ATPase.

An afebrile 22 year-old female presents with 2 days of a painful rectal mass which worsens with defecation. She denies any blood with bowel movements. She admits to ongoing constipation and generally poor diet of fast food. She is otherwise healthy. On examination there is a tender, swollen, bluish, ovoid mass visible at the anal margin. Which of the following is the most likely diagnosis? A. Rectal prolapse B. Thrombosed hemorrhoid C. Internal Hemorrhoid D. Rectal polyp

(u) A. Rectal prolapse shows radiating folds and is typically painless. (c) B. A thrombosed hemorrhoid causes acute pain increased with defecation and sitting. (u) C. Internal hemorrhoids are an enlargement of the normal vascular cushions located above the pectinate line. They are not usually palpable and might cause bright red bleeding during defecation. (u) D. Polyps are soft and may be difficult or impossible to feel. Proctoscopy is usually required for diagnosis.

Which of the following typical findings would be revealed during a sigmoidoscopy on a patient with Crohn's disease of the intestine? A. Rectal pseudopolyps B. Diffuse ulceration and bleeding C. Sheets of WBCs with inflamed mucosa D. Intermittent longitudinal mucosal ulcers and fissures

(u) A. Rectal pseudopolyps are associated with ulcerative colitis rather than Crohn's. (u) B. Diffuse ulcerations and bleeding are more characteristic of ulcerative colitis than Crohn's disease. (u) C. Sheets of WBCs or "pseudomembranes" can be detected in patients with pseudomembranous colitis. (c) D. Ulcerations tend to be linear with transverse fissures in Crohn's disease. These skip lesions are common with Crohn's disease.

In a patient with metabolic acidosis due to ingestion of methanol, which substance below is considered therapeutic? A. Salicylates B. Paraldehyde C. Ethylene glycol D. Ethanol

(u) A. Salicylates, paraldehyde, ethylene glycol all cause metabolic acidosis with an elevated anion gap. (u) B. See A for explanation. (u) C. See A for explanation. (c) D. Ethanol, along with hemodialysis and supportive measures, is indicated for metabolic acidosis caused by methanol ingestion.

A 42 year-old patient who is being treated for colon cancer with chemotherapy develops nausea and vomiting. Which of the following drugs would be the most effective in controlling the nausea and vomiting? A. scopolamine (Scopace) B. meclizine (Antivert) C. ondansetron (Zofran) D. loperamide (Imodium)

(u) A. Scopolamine and meclizine are effective against motion sickness, but ineffective against substances that act directly on the chemoreceptor trigger zone. (u) B. See A for explanation. (c) C. Ondansetron selectively blocks 5-HT3 receptors in the periphery (visceral afferent fibers) and in the brain (chemoreceptor trigger zone). It is indicated for use in chemotherapy induced nausea and vomiting. (u) D. Loperamide has low anti-emetic potency.

A 72 year-old woman presents to your clinic complaining of constipation. Which of the following presentations would be most concerning? A. Fewer than one stools per week B. Weight loss C. Recurrent hemorrhoids D. Scybala

(u) A. See B for explanation (c) B. Alarm symptoms are concerning for colorectal malignancy and include weight loss, anemia, hematochezia, or positive fecal occult blood test. These symptoms are particularly worrisome in a patient with a family history of colorectal cancer. (u) C. See B for explanation. (u) D. See B for explanation.

A 42 year-old male with chronic complaints of heartburn and regurgitation presents for follow-up after undergoing endoscopic evaluation which shows evidence of Barrett's esophagus. Which of the following is the most serious potential complication in this patient? A. Peptic stricture B. Esophageal adenocarcinoma C. Esophageal web formation D. Mallory-Weiss tear

(u) A. See B for explanation. (c) B. Esophageal carcinoma is the most serious complication of Barrett's esophagus. (u) C. See B for explanation. (u) D. See B for explanation.

An asymptomatic 30 year-old male with no significant past medical history presents to the office. He is concerned about his father being diagnosed with colon cancer at the age of 60. Which of the following is the most appropriate health counseling for this patient? A. Flexible sigmoidoscopy now B. Colonoscopy at age 40 C. Flexible sigmoidoscopy age 35 D. Colonoscopy age 50

(u) A. See B for explanation. (c) B. It is recommended that patients with a single first-degree relative with colon cancer diagnosed at or after age 60 should begin screening at age 40 preferably with a colonoscopy. (u) C. See B for explanation. (u) D. See B for explanation

A 22 year-old graduate student presents with a six-month history of abdominal pain relieved with defecation which seems to coincide with her starting PA school. She describes alternating constipation and diarrhea as well as bloating. She denies any recent weight changes. Her only medication is citalopram (Celexa) What is the most likely diagnosis for this patient? A. Crohn's disease B. Irritable bowel syndrome C. Thyroid storm D. Celiac disease

(u) A. See B for explanation. (c) B. The patient's symptoms are consistent with irritable bowel syndrome. More than 50% of patients who seek medical attention for symptoms of IBS also have a diagnosis of depression, anxiety, or somatization. (u) C. See B for explanation. (u) D. See B for explanation.

A 65 year-old woman is admitted to the hospital for severe abdominal pain and vomiting over the last 6 hours. Her laboratory findings WBC Count - 19,000 Serum triglyceride level - 1100 mg/dL Glucose - 280 mg/dL Which of the following is the most likely diagnosis? A. Acute cholecystitis B. Acute pancreatitis C. Hepatitis D. Diabetic ketoacidosis

(u) A. See B for explanation. (c) B. This patient has acute pancreatitis. Typical symptoms include abdominal pain and nausea and vomiting. Significant hypertriglyceridemia is present. Transient hyperglycemia is common, in addition to elevation of lipase and amylase, serum amino-transferase and alkaline phosphatase are elevated in 50% of patients with acute pancreatitis. (u) C. See B for explanation. (u) D. See B for explanation.

A cirrhotic patient presents with progressive drowsiness and delirium. Physical examination reveals asterixis and tremor. Which of the following medications would you initiate? A. Empiric antibiotic therapy B. Lactulose (Kristalose) C. Bolus fluid challenge with sodium chloride D. Hydrochlorothiazide (Diuril)

(u) A. See B for explanation. (c) B. This patient has hepatic encephalopathy. Increased ammonia levels contribute to the mental status changes associated with hepatic encephalopathy. Lactulose leads to a change in bowel flora so that fewer ammonia forming organisms are present and also helps in the acidification of colon contents which leads to a nonabsorbable ammonium ion creation. (u) C. See B for explanation. (u) D. See B for explanation.

An elderly man who recently emigrated from a war-torn area of Africa is brought to the clinic by his daughter. She explains that her father's diet was very limited in calories and protein and that he mostly ate corn and very little fresh foods. He has chronic diarrhea and examination reveals pigmented regions on hands, arms, and face and mild dementia. His tongue is smooth and shiny. This patient most likely has a deficiency of which of following vitamins? A. Folate B. Niacin C. Thiamine D. Vitamin K

(u) A. See B for explanation. (c) B. This patient's syndrome known as pellagra is due to niacin (vitamin B3) deficiency. It is often seen in people where corn is the major source of energy and is still endemic in parts of Africa. The syndrome includes glossitis, pigmented dermatitis, dementia, and diarrhea. (u) C. See B for explanation. (u) D. See B for explanation.

Which of the following is the study of choice to diagnose upper gastrointestinal malignancy? A. Abdominal CT B. Upper endoscopy C. Barium swallow D. Abdominal ultrasound

(u) A. See B for explanation. (c) B. Upper endoscopy is the study of choice to diagnose gastroduodenal ulcers, erosive esophagitis and upper gastrointestinal malignancy. (u) C. See B for explanation. (u) D. See B for explanation.

What physical examination finding is most specific for acute cholecystitis? A. Psoas sign B. Rovsing's sign C. Murphy's sign D. Cullen's sign

(u) A. See C for explanation. (u) B. See C for explanation. (c) C. A sharp increase in tenderness with a sudden stop in inspiratory effort constitutes a postive Murphy's sign and is most specific for cholecystitis. (u) D. See C for explanation.

A newborn weighs 8 pounds at birth. On average, what should the infant weigh at 1 year of age? A. 16 pounds B. 20 pounds C. 24 pounds D. 28 pounds

(u) A. See C for explanation. (u) B. See C for explanation. (c) C. An infant will triple birth weight within the first year of life. A newborn that weighs 8 pounds at birth will weigh approximately 24 pounds at 1 year of age. (u) D. See C for explanation.

At birth an infant weighs 8 pounds. The optimal weight for this infant at 1 year would be A. 16 pounds. B. 20 pounds. C. 24 pounds. D. 28 pounds.

(u) A. See C for explanation. (u) B. See C for explanation. (c) C. An infants birth weight should double by 4-5 months of age and triple by 1 year of age. (u) D. See C for explanation.

A 43 year-old female calls the office stating that her husband was recently diagnosed with Hepatitis A. She denies a past history of or immunization for this disease and specifically denies recent jaundice, fever, nausea, or abdominal pain. Which of the following is the most appropriate next step in the care of this patient? A. Prednisone B. Ribavirin (Virazole) C. Immune globulin D. Lamivudine (Epivir)

(u) A. See C for explanation. (u) B. See C for explanation. (c) C. Immune globulin is indicated in close contacts of patients with Hepatitis A who have not previously had the disease or been immunized against it. (u) D. See C for explanation.

An asymptomatic 50 year-old person has no risk factors for colorectal cancer. In addition to yearly hemoccult screening of the stool, how often should flexible sigmoidoscopy be performed? A. Every year B. Every other year C. Every 5 years D. Every 10 years

(u) A. See C for explanation. (u) B. See C for explanation. (c) C. In a person with no risk factors for colorectal cancer, screening should include annual Hemoccult testing and flexible sigmoidoscopy every 5 years. (u) D. See C for explanation.

A 34 year-old female 3 days status-post appendectomy complains of mild diffuse abdominal pain with associated nausea and vomiting. On examination you note diminished bowel sounds. There is generalized abdominal distension with mild tenderness on palpation. There are no signs of peritoneal inflammation. Abdominal radiographs show distended gas-filled loops of the small and large intestine. Abdominal CT scan shows no mechanical obstruction. Which of the following is the most appropriate management for this patient? A. Docusate sodium (Colace) B. Colonoscopic decompression C. Restrict oral intake D. Exploratory laparoscopy

(u) A. See C for explanation. (u) B. See C for explanation. (c) C. Most adynamic ileuses respond to restriction of oral intake and gradual advancement of diet as bowel function returns. (u) D. See C for explanation.

A 27 year-old female is brought to the hospital after being found unconscious with a half-empty bottle of Hydrocodone/Acetominophen (Vicodin) in her hand. Which of the following is the most appropriate treatment for this patient? A. Flumazenil (Romazicon) B. Disulfiram (Antabuse) C. Naloxone (Narcan) D. Activated charcoal

(u) A. See C for explanation. (u) B. See C for explanation. (c) C. Naloxone is the antidote for opioid ingestion. (u) D. See C for explanation.

As a rule, solid foods such as cereal and fruits are best introduced into an infant's diet at approximately A. 1 to 3 weeks. B. 4 to 8 weeks. C. 4 to 6 months. D. 10 to 14 months.

(u) A. See C for explanation. (u) B. See C for explanation. (c) C. The inclusion of solid foods in the diet is best done at approximately 4-6 months. Before 4 to 6 months of age does not contribute significantly to the infant's health and it increases their risk of allergies and atopy. (u) D. After 6 months, meat, eggs, and starchy foods can be introduced.

A 50 year-old female presents with constipation following an episode of tearing pain associated with bleeding while defecating 4 days ago. She admits to a residual throbbing pain and she is afraid to have a bowel movement. On examination you note what appears to be a crack in the epithelium of the anal verge. Which of the following is the most appropriate initial management for this patient? A. Anal sphincter strengthening exercises B. Injection sclerotherapy C. Fiber supplementation and sitz baths D. Analgesics with incision and drainage

(u) A. See C for explanation. (u) B. See C for explanation. (c) C. This patient has signs and symptoms consistent with an anal fissure. Most patients will respond to fiber supplementation and sitz baths. (u) D. See C for explanation.

A patient presents to the emergency department with right upper quadrant pain over eight hours, nausea, and vomiting. On exam there is a fever of 101.2 degrees F. Ultrasound shows a distended gallbladder. What is the most appropriate management of this patient? A. Oral analgesics B. Diagnostic peritoneal lavage C. Proton pump inhibitors D. Laparoscopic cholecystectomy

(u) A. See D for explanation. (h) B. Diagnostic peritoneal lavage is used to detect intraabdominal bleeding from trauma and not to treat acute cholecystitis and may delay appropriate treatment. (u) C. Proton pump inhibitors are used to treat GERD or PUD. (c) D. The proper treatment for acute cholecystitis is IV fluids, antibiotics, pain control, and surgery. Cholecystectomy is the definitive treatment for acute cholecystitis and laparoscopic cholecystectomy is the procedure of choice.

The birth weight of an infant has usually tripled by A. the second and third month. B. the fourth and fifth month. C. the seventh and eighth month. D. one year.

(u) A. See D for explanation. (u) B. See D for explanation. (u) C. See D for explanation. (c) D. A child triples his/her birthweight by one year.

A 41 year-old female presents with RUQ pain. She is febrile and slightly tachycardic. Examination demonstrates abdominal tenderness and abrupt cessation of inspiration with palpation under the right costal margin during deep inspiration. Ingestion of which of the following is most likely to have proceeded the onset of this condition? A. Alcoholic beverage B. Dairy product C. High-fiber meal D. Fatty meal

(u) A. See D for explanation. (u) B. See D for explanation. (u) C. See D for explanation. (c) D. Cholecystitis presents with complaint of RUQ pain and on exam there may be a positive Murphy's sign. It is associated with meals high in fat. Alcohol, dairy products and high fiber meals are not associated with cholecystitis and would not present as fever, tenderness and positive Murphy's sign

A 41 year-old female presents to you for medical screening advice. Her 44 year-old sister passed away recently 18 months after diagnosis of metastatic colon cancer. Which of the following is the most appropriate advice for this patient? A. Double contrast barium enema now and repeat every 5 years if normal B. Rectal occult blood testing annually until age 50 then sigmoidoscopy every 3 years C. Rectal occult blood testing annually until age 50 then screening colonoscopy every 5 years D. Screening colonoscopy now and repeat every 3-5 years if normal

(u) A. See D for explanation. (u) B. See D for explanation. (u) C. See D for explanation. (c) D. Hereditary factors are believed to contribute to up to 30% of colorectal cancers. Relative risk is 3.8 times if the family member's cancer was diagnosed at less than 45 years of age. Recommended screening in a single first degree relative with colorectal cancer diagnosed before age 60 is beginning colonoscopy at age 40 or ten years younger than age at diagnosis of youngest affected first-degree relative. Then if negative, every 5 years.

A 10 year-old African immigrant presents 2 weeks after arriving in the US describing dry eyes and difficulty with night vision. Examination reveals numerous, small white conjunctival patches bilaterally. This patient's symptoms are most likely due to which of the following deficiencies? A. Vitamin B6 B. Vitamin C C. Vitamin D D. Vitamin A

(u) A. See D for explanation. (u) B. See D for explanation. (u) C. See D for explanation. (c) D. Vitamin A deficiency is one of the most common causes of blindness in developing countries. It is also seen secondary to malabsorption disorders and frequent mineral oil laxative use. Night blindness is one of the earliest symptoms.

Prolonged use of a proton pump inhibitor can lead to low levels of which of the following nutrients? A. Folic acid B. Magnesium C. Vitamin B6 D. Vitamin B12

(u) A. See D for explanation. (u) B. See D for explanation. (u) C. See D for explanation. (c) D. Vitamin B12 requires gastric acid for absorption in the stomach. Prolonged use of a proton pump inhibitor suppresses gastric acid production.

A severely dehydrated child with gastroenteritis who is unable to tolerate oral rehydration should receive which of the following intravenous therapies? A. 10 mg/kg normal saline B. 20 mg/kg normal saline C. 10 mg/kg D5W D. 20 mg/kg D5W

(u) A. See answer B for explanation. (c) B. In a severely dehydrated child, restoring intravascular volume to insure adequate tissue perfusion is the immediate objective. This is best done with either Ringers lactate or normal saline. The addition of potassium would only be done after initial fluid boluses and after insuring adequate kidney function. The addition of glucose to the IV solution may result in an osmotic diuresis worsening the dehydration. (u) C. See answer B for explanation. (u) D. See answer B for explanation

Which of the following is used to screen for malabsorption disorders of the intestines? A. serum gastrin level B. urea breath test C. fecal eosinophils D. stool fecal fat

(u) A. Serum gastric level is elevated in Zollinger-Ellison syndrome. (u) B. Urea breath test is used in the diagnosis of H. pylori infection. (u) C. Eosinophils are elevated in inflammatory diarrhea due to eosinophilic gastroenteritis. (c) D. Stool fecal fat is the gold standard test for the evaluation of patients with malabsorption of the intestine.

A patient diagnosed with Barrett's esophagus is at an increased risk for the development of what type of cancer? A. Squamous cell B. Transitional cell C. Adenocarcinoma D. Atypical carcinoid

(u) A. Squamous cell is not typical for esophageal cancer. (u) B. Transitional cell is a cancer of the bladder. (c) C. The most serious complication of Barrett's esophagus is esophageal adenocarcinoma. (u) D. Atypical carcinoid is more typical of lung cancer.

A patient develops abdominal cramps and watery diarrhea 10 to 12 hours after eating a plate of unrefrigerated meat and vegetables. The patient denies vomiting. The causative agent is most likely A. Staphylococcus aureus. B. Clostridium perfringens. C. Escherichia coli. D. Salmonella.

(u) A. Staphylococcus aureus has an incubation period of 1 to 6 hours and also characteristically has vomiting associated with it. (c) B. Food poisoning caused by Clostridium perfringens has an incubation period of 8 to 14 hours and results frompoorly refrigerated cooked meat. (u) C. The incubation period for Escherichia coli and Salmonella is greater than 16 hours. (u) D. See C for explanation.

A 32 year-old presents with a 3-day history of diarrhea. The patient denies blood, mucus, or night awakening with diarrhea. He recently returned from a business trip to Canada. On physical examination, the patient is afebrile and vital signs reveal BP 115/80, pulse is 76, and respirations are 14. The abdominal examination reveals hyperactive bowel sounds, but is otherwise unremarkable. Which of the following is the most appropriate initial intervention? A. Stool for culture, ova, and parasites B. Proctosigmoidoscopy C. Metronidazole (Flagyl) D. Supportive treatment

(u) A. Stool culture and examination for ova and parasites are indicated when the diarrhea has persisted longer than 3 weeks or is associated with abdominal pain, fever, and/or bloody stools. (u) B. Proctosigmoidoscopy is indicated when inflammatory bowel disease is suspected on the basis of fever, bloody diarrhea, or abdominal pain. (u) C. Metronidazole is indicated with a confirmed diagnosis of Giardia lamblia or amebic disease. (c) D. Symptomatic treatment, including dietary management and over-the-counter antidiarrheals, is indicated for afebrile patients with watery diarrhea of less than 5 days duration.

The main complication with the use of transjugular intrahepatic portosystemic shunt (TIPS) procedure is which of the following? A. increased portal pressures resulting in further esophageal varices B. increased portal pressures resulting in a worsening of cirrhosis C. Budd-Chiari syndrome D. increased risk of encephalopathy

(u) A. TIPS procedures are performed in order to shunt blood away from the liver parenchyma which in essence lowers portal pressures lessening the risk for esophageal varices. (u) B. TIPS procedures, since they cause of bypass of the liver parenchyma, result in a lessening of the blood flow into the liver which does not cause cirrhosis to progress. (u) C. Budd-Chiari syndrome is a thrombosis of the hepatic vein. It is not a common complication of the TIPS procedure. (c) D. TIPS procedures involve the placement of a stent in the liver in order to shunt blood away from the portal vein into the hepatic vein which bypasses the cirrhotic liver parenchyma. Its main complication is encephalopathy from the accumulation of toxic substances in the brain since the liver no longer acts as a filter.

A middle-aged patient is being treated for recurrent diarrhea and peptic ulcer disease that is refractory adequate standard therapy. Which of the following is the most likely diagnosis? A. Achlorhydria B. Drug resistant H. pylori infection C. Zollinger-Ellison syndrome D. Giardiasis

(u) A. The clinical picture of diarrhea with peptic ulcer disease refractory to adequate standard therapy is consistent with gastric acid hypersecretion, not achlorhydria. (u) B. Drug resistant infection from H. pylori has not been reported and would not explain the associated diarrhea. (c) C. Zollinger-Ellison syndrome is the result of unregulated release of gastrin resulting in gastric acid hypersecretion. Up to 50% of patients complain of diarrhea along with peptic ulcer disease. (u) D. Giardiasis causes acute diarrhea but is not associated with peptic ulcer disease.

A 32 year-old female presents with an 8-week history of a burning sensation deep in the chest, which starts about 30 minutes after she eats and worsens when she lays down at night. She has had some relief with an antacid and denies dysphagia. Which of the following interventions would be beneficial for this patient? A. Raising the foot of the bed B. Taking Cimetidine (Tagamet) 30 minutes after a meal C. Eating 3 large meals per day D. Avoiding mint

(u) A. The head of the bed should be raised 6 inches to allow gravity to decrease reflux. (u) B. H2 blockers have a 30-minute onset of action and so are best taken 30 minutes before a meal known to cause reflux. (u) C. Patients usually do better with smaller, more frequent meals, along with the recommendation to avoid eating 3 hours before bedtime. (c) D. Both peppermint and chocolate are substances known to aggravate reflux.

Which of the following is the mechanism of action of infliximab (Remicade)? A. block lymphocyte proliferation B. direct effect on smooth muscle C. monoclonal antibody that binds to tumor necrosis factor D. 5-HT antagonist

(u) A. The immunomodulators (6-MP) work by blocking lymphocyte proliferation. (u) B. The mechanism of action of mebeverine is direct effect on smooth muscle in irritable bowel syndrome. (c) C. Infliximab is a monoclonal antibody that binds to tumor necrosis factor. (u) D. The anti-nausea and vomiting medications like ondansetron work by blocking 5-HT receptors.

A 45 year-old type 2 diabetic female with history of cholelithiasis presents to the clinic with 2-3 episodes of sudden, severe epigastric pain that radiates to her shoulder. She has associated nausea and vomiting. Temperature is 101 degrees F and she is experiencing chills. Today her eyes appear yellow in color. Which of the following is the most likely diagnosis for this patient A. Postcholecystectomy syndrome B. Cholangitis C. Gastroesophageal reflux disease D. Pancreatic cancer

(u) A. The patient has no history of previous gall bladder surgery. (c) B. Cholangitis is characterized by a history of biliary pain, fever, chills, and jaundice associated with episodes of abdominal pain. (u) C. Gastroesophageal reflux disease (GERD) is characterized by heartburn. Fever and jaundice are not typical features of GERD. (u) D. Pancreatic cancer, although a possibility, is characterized by chronic weight loss, epigastric pain radiating to the back, and occasional jaundice. Fever and chills are not typical features.

A 25 year-old woman complains of bloating, abdominal cramps, and loose stools with flatulence. Her symptoms seem to occur 1-2 hours after meals containing any dairy products. Which of the following would be the most appropriate diagnostic test to confirm your suspicion? A. Colonoscopy B. Fecal smear for leukocytes C. Hydrogen breath test D. Serum IgA endomysial antibody

(u) A. The symptoms are due to a malabsorption of a selective carbohydrate (lactose) in the small intestine. Colonoscopy will not elucidate the cause. (u) B. Fecal leukocytes indicate diffuse colonic inflammation and is ordered in more moderate or severe cases of infectious diarrhea (bacterial). This patient's symptoms are associated with particular enzyme deficiency. (c) C. This woman most likely has lactase deficiency as her symptoms occur after dairy ingestion. Hydrogen breath test after administration of lactose will confirm the diagnosis. (u) D. Serum EMA IgA antibody is a screening test for gluten-sensitive enteropathy; symptoms occur due to ingestion of wheat not dairy products

Which of the following treatments is the most appropriate for a patient with an acutely tender, fluctuant perirectal mass? A. Fistulectomy B. Drainage of an abscess C. Antibiotic therapy D. Removal of an anal fissure

(u) A. There is no fistula present; therefore, a fistulectomy is not indicated. (c) B. Perirectal abscesses should be treated by drainage as soon as a diagnosis is established. (u) C. Antibiotic therapy is not indicated unless there is extensive cellulitis, or the patient is immunocompromised, has diabetes mellitus, or has valvular heart disease. (u) D. Anal fissures are linear tears. Indications for operative management include persistent pain, lack of healing, and recurrence.

Which of the following can be a very serious consequence of using antidiarrheals in a patient with inflammatory bowel disease? A. Lymphoma B. Toxic megacolon C. Bone marrow suppression D. Delayed serum sickness-like reaction

(u) A. There is suggestion that lymphoma may result from the use of infliximab, an anti-TNF agent, utilized in refractory cases of Crohn's disease; however a clear link between the development of lymphoma and the use of infliximab has not been established. (c) B. Antidiarrheals may cause the development of toxic megacolon when used by patients with active severe inflammatory bowel disease. (u) C. Bone marrow suppression is a side effect of methotrexate, which is being increasingly used in the treatment of Crohn's disease. (u) D. Delayed serum sickness-like reaction is another potential side effect of infliximab.

A patient is hospitalized with a change in mental status. Examination reveals that he is unable to maintain dorsiflexion of the wrists after pronating his arms in front of his body. Which of the following is the most likely diagnosis? A. cocaine overdose B. hyperthyroidism C. hepatic encephalopathy D. Parkinson's disease

(u) A. Tremor and agitation are part of acute cocaine intoxication, not asterixis. (u) B. Hyperthyroidism causes a fine resting tremor, not asterixis. (c) C. This is the description for asterixis that is seen with hepatic encephalopathy, uremia, and carbon dioxide narcosis. (u) D. Parkinson's disease has resting tremor, rigidity, akinesia, and postural hypotension, not asterixis.

Which of the following studies is most appropriate to diagnose celiac disease? A. Urinary D-xylose test B. Small bowel biopsy C. Barium contrast x-ray D. Schilling test

(u) A. Urinary D-xylose test assesses the proximal small intestinal mucosa's absorption of carbohydrates but is not specific to celiac disease. (c) B. Definitive diagnosis of celiac disease is made by small bowel biopsy. (u) C. Barium contrast examination would reveal anatomic abnormalities or blind loop syndrome that may lead to malabsorption but it would not help in the diagnosis of celiac disease. (u) D. The Schilling test is used to determine the cause of cobalamine malabsorption from pernicious anemia, chronic pancreatitis, achlorhydria and bacterial overgrowth syndrome but has no role in the diagnosis of celiac disease.

A 62 year-old male presents with complaints of vague epigastric abdominal pain associated with jaundice and generalized pruritus. Physical examination reveals jaundice and a palpable non-tender gallbladder, but is otherwise unremarkable. Which of the following is the most likely diagnosis? A. Viral hepatitis B. Pancreatic cancer C. Acute cholecystitis D. Gilbert's syndrome

(u) A. While viral hepatitis may cause jaundice, the liver is enlarged and tender. (c) B. Pancreatic cancer is suggested by the vague epigastric pain with the jaundice resulting from biliary obstruction due to cancer involving the pancreatic head. The presence of a palpable non-tender gallbladder (Courvoisier's sign) also indicates obstruction due to the cancer. (u) C. While acute cholecystitis may present with jaundice and an enlarged gallbladder, the pain is classically colicky and located in the right upper quadrant. On physical examination with deep inspiration and palpation of the right subcostal area increased pain and respiratory arrest (Murphy's sign) is usually seen. (u) D. Gilbert's syndrome is the most common of the hereditary hyperbilirubinemias. It is most often diagnosed near puberty or adult life based on results of a comprehensive metabolic panel.

Which of the following is the most common indication for operative intervention in patients with chronic pancreatitis? A.Weight loss B. Intractable pain C. Exocrine deficiency D. To decrease risk of cancer

(u) A. While weight loss is common with chronic pancreatitis, it is not an indication for surgical intervention. (c) B. Indications for surgical treatment of chronic pancreatitis include severe pain that limits the patient's functioning or intractable pain despite the use of non-narcotic analgesics and absence of alcohol intake. (u) C. While the majority of patients go on to develop diabetes mellitus 25 years after the clinical onset of chronic pancreatitis, this is not an indication for surgical intervention as it would lead to more severe exocrine deficiency. (u) D. While the possible presence of pancreatic cancer is an indication for surgery, there is no indication for prophylactic surgery to decrease the risk of cancer.

What is the initial treatment of choice for inflammatory bowel disease? A. Ondansetron (Zofran) B. Sulfasalazine (Azulfidine) C. Metronidazole (Flagyl) D. Azothioprine (Imuran)

(u) A. Zofran is an antiemetic and not indicated in the treatment of inflammatory bowel disease. (c) B. Sulfasalazine and other 5-aminosalicylic acid drugs are the cornerstone of therapy in mild to moderate inflammatory bowel disease as they have both anti-inflammatory and antibacterial properties. (u) C. Metronidazole may have a role in Crohn's disease after ileal resection but has no role in the treatment of mild to moderate disease. (u) D. Azothioprine is used in severe, glucocorticoid-dependent inflammatory bowel disease.


Ensembles d'études connexes

Hashing and Digital Certificates

View Set

Parts of Speech Review: Nouns, Adjectives, Verbs, Adverbs and Prepositions

View Set

PPR. COMPETENCY 006. CLASSROOM MANAGEMENT. Strategies for creating an organized and productive learning environment and for managing student behavior.

View Set

Driver's Ed Alcohol and Other Drugs

View Set

chapter 11 / lesson 3 - lifelong nutrition

View Set